Programs & Examples On #Apache2 module

0

How to show all of columns name on pandas dataframe?

The easiest way I've found is just

list(df.columns)

Personally I wouldn't want to change the globals, it's not that often I want to see all the columns names.

How do I turn off Oracle password expiration?

To alter the password expiry policy for a certain user profile in Oracle first check which profile the user is using:

select profile from DBA_USERS where username = '<username>';

Then you can change the limit to never expire using:

alter profile <profile_name> limit password_life_time UNLIMITED;

If you want to previously check the limit you may use:

select resource_name,limit from dba_profiles where profile='<profile_name>';

Java: Add elements to arraylist with FOR loop where element name has increasing number

You can't do it the way you're trying to... can you perhaps do something like this:

List<Answer> answers = new ArrayList<Answer>();
for(int i=0; i < 4; i++){
  Answer temp = new Answer();
  //do whatever initialization you need here
  answers.add(temp);
}

Could not find the main class, program will exit

The classpath is the path that the system will follow when trying to find the classes that you're trying to run. In the batch file you're trying to execute it probably has a variable like CLASSPATH=blah;blah;etc or a java command that looks similar to

java -classpath "c:\directory\lib\squirrel-sql.jar" com.some.squirrel.package.file

If you can find or add that classpath setting, make sure that it includes a path to the squirrel-sql.jar and any other jar files that it may depend on separated by semicolons (or the root /lib directory that may be included with the installation).

Basically you just need to tell java where to find the class files that you're trying to execute. Wikipedia has a more indepth discussion about classpath and can offer you more insight. http://en.wikipedia.org/wiki/Classpath_(Java)

Ternary operator in PowerShell

I've recently improved (open PullRequest) the ternary conditional and null-coalescing operators in the PoweShell lib 'Pscx'
Pls have a look for my solution.


My github topic branch: UtilityModule_Invoke-Operators

Functions:

Invoke-Ternary
Invoke-TernaryAsPipe
Invoke-NullCoalescing
NullCoalescingAsPipe

Aliases

Set-Alias :?:   Pscx\Invoke-Ternary                     -Description "PSCX alias"
Set-Alias ?:    Pscx\Invoke-TernaryAsPipe               -Description "PSCX alias"
Set-Alias :??   Pscx\Invoke-NullCoalescing              -Description "PSCX alias"
Set-Alias ??    Pscx\Invoke-NullCoalescingAsPipe        -Description "PSCX alias"

Usage

<condition_expression> |?: <true_expression> <false_expression>

<variable_expression> |?? <alternate_expression>

As expression you can pass:
$null, a literal, a variable, an 'external' expression ($b -eq 4) or a scriptblock {$b -eq 4}

If a variable in the variable expression is $null or not existing, the alternate expression is evaluated as output.

Keeping it simple and how to do multiple CTE in a query

You can have multiple CTEs in one query, as well as reuse a CTE:

WITH    cte1 AS
        (
        SELECT  1 AS id
        ),
        cte2 AS
        (
        SELECT  2 AS id
        )
SELECT  *
FROM    cte1
UNION ALL
SELECT  *
FROM    cte2
UNION ALL
SELECT  *
FROM    cte1

Note, however, that SQL Server may reevaluate the CTE each time it is accessed, so if you are using values like RAND(), NEWID() etc., they may change between the CTE calls.

How to convert Json array to list of objects in c#

Did you check this line works perfectly & your string have value in it ?

string jsonString = sr.ReadToEnd();

if yes, try this code for last line:

ValueSet items = JsonConvert.DeserializeObject<ValueSet>(jsonString);

or if you have an array of json you can use list like this :

List<ValueSet> items = JsonConvert.DeserializeObject<List<ValueSet>>(jsonString);

good luck

How to use SVG markers in Google Maps API v3

Yes you can use an .svg file for the icon just like you can .png or another image file format. Just set the url of the icon to the directory where the .svg file is located. For example:

var icon = {
        url: 'path/to/images/car.svg',
        size: new google.maps.Size(sizeX, sizeY),
        origin: new google.maps.Point(0, 0),
        anchor: new google.maps.Point(sizeX/2, sizeY/2)
};

var marker = new google.maps.Marker({
        position: event.latLng,
        map: map,
        draggable: false,
        icon: icon
});

How to submit an HTML form on loading the page?

You don't need Jquery here! The simplest solution here is (based on the answer from charles):

<html>
<body onload="document.frm1.submit()">
   <form action="http://www.google.com" name="frm1">
      <input type="hidden" name="q" value="Hello world" />
   </form>
</body>
</html>

Fitting a histogram with python

Here you have an example working on py2.6 and py3.2:

from scipy.stats import norm
import matplotlib.mlab as mlab
import matplotlib.pyplot as plt

# read data from a text file. One number per line
arch = "test/Log(2)_ACRatio.txt"
datos = []
for item in open(arch,'r'):
    item = item.strip()
    if item != '':
        try:
            datos.append(float(item))
        except ValueError:
            pass

# best fit of data
(mu, sigma) = norm.fit(datos)

# the histogram of the data
n, bins, patches = plt.hist(datos, 60, normed=1, facecolor='green', alpha=0.75)

# add a 'best fit' line
y = mlab.normpdf( bins, mu, sigma)
l = plt.plot(bins, y, 'r--', linewidth=2)

#plot
plt.xlabel('Smarts')
plt.ylabel('Probability')
plt.title(r'$\mathrm{Histogram\ of\ IQ:}\ \mu=%.3f,\ \sigma=%.3f$' %(mu, sigma))
plt.grid(True)

plt.show()

enter image description here

SSL_connect: SSL_ERROR_SYSCALL in connection to github.com:443

A simple restart fixed it for me. I'm not sure what was the problem since I work with so much software but I have a feeling it was the VPN software or maybe the fact I put my laptop in sleep a lot and some file was corrupted. I really don't know but the restart fixed it.

Copy a git repo without history

Isn't this exactly what squashing a rebase does? Just squash everything except the last commit and then (force) push it.

Dockerfile copy keep subdirectory structure

Alternatively you can use a "." instead of *, as this will take all the files in the working directory, include the folders and subfolders:

FROM ubuntu
COPY . /
RUN ls -la /

Can regular JavaScript be mixed with jQuery?

You can, but be aware of the return types with jQuery functions. jQuery won't always use the exact same JavaScript object type, although generally they will return subclasses of what you would expect to be returned from a similar JavaScript function.

How do you run your own code alongside Tkinter's event loop?

When writing your own loop, as in the simulation (I assume), you need to call the update function which does what the mainloop does: updates the window with your changes, but you do it in your loop.

def task():
   # do something
   root.update()

while 1:
   task()  

Anonymous method in Invoke call

I had problems with the other suggestions because I want to sometimes return values from my methods. If you try to use MethodInvoker with return values it doesn't seem to like it. So the solution I use is like this (very happy to hear a way to make this more succinct - I'm using c#.net 2.0):

    // Create delegates for the different return types needed.
    private delegate void VoidDelegate();
    private delegate Boolean ReturnBooleanDelegate();
    private delegate Hashtable ReturnHashtableDelegate();

    // Now use the delegates and the delegate() keyword to create 
    // an anonymous method as required

    // Here a case where there's no value returned:
    public void SetTitle(string title)
    {
        myWindow.Invoke(new VoidDelegate(delegate()
        {
            myWindow.Text = title;
        }));
    }

    // Here's an example of a value being returned
    public Hashtable CurrentlyLoadedDocs()
    {
        return (Hashtable)myWindow.Invoke(new ReturnHashtableDelegate(delegate()
        {
            return myWindow.CurrentlyLoadedDocs;
        }));
    }

Removing multiple keys from a dictionary safely

Some timing tests for cpython 3 shows that a simple for loop is the fastest way, and it's quite readable. Adding in a function doesn't cause much overhead either:

timeit results (10k iterations):

  • all(x.pop(v) for v in r) # 0.85
  • all(map(x.pop, r)) # 0.60
  • list(map(x.pop, r)) # 0.70
  • all(map(x.__delitem__, r)) # 0.44
  • del_all(x, r) # 0.40
  • <inline for loop>(x, r) # 0.35
def del_all(mapping, to_remove):
      """Remove list of elements from mapping."""
      for key in to_remove:
          del mapping[key]

For small iterations, doing that 'inline' was a bit faster, because of the overhead of the function call. But del_all is lint-safe, reusable, and faster than all the python comprehension and mapping constructs.

How to retrieve the current version of a MySQL database management system (DBMS)?

You can also look at the top of the MySQL shell when you first log in. It actually shows the version right there.

Welcome to the MySQL monitor.  Commands end with ; or \g.
Your MySQL connection id is 67971
Server version: 5.1.73 Source distribution

Copyright (c) 2000, 2013, Oracle and/or its affiliates. All rights reserved.

Oracle is a registered trademark of Oracle Corporation and/or its
affiliates. Other names may be trademarks of their respective
owners.

Type 'help;' or '\h' for help. Type '\c' to clear the current input statement.

mysql>

Access Control Origin Header error using Axios in React Web throwing error in Chrome

If your backend support CORS, you probably need to add to your request this header:

headers: {"Access-Control-Allow-Origin": "*"}

[Update] Access-Control-Allow-Origin is a response header - so in order to enable CORS - you need to add this header to the response from your server.

But for the most cases better solution would be configuring the reverse proxy, so that your server would be able to redirect requests from the frontend to backend, without enabling CORS.

You can find documentation about CORS mechanism here: https://developer.mozilla.org/en-US/docs/Web/HTTP/Access_control_CORS

NSRange to Range<String.Index>

A riff on the great answer by @Emilie, not a replacement/competing answer.
(Xcode6-Beta5)

var original    = "This is a test"
var replacement = "!"

var startIndex = advance(original.startIndex, 1) // Start at the second character
var endIndex   = advance(startIndex, 2) // point ahead two characters
var range      = Range(start:startIndex, end:endIndex)
var final = original.stringByReplacingCharactersInRange(range, withString:replacement)

println("start index: \(startIndex)")
println("end index:   \(endIndex)")
println("range:       \(range)")
println("original:    \(original)")
println("final:       \(final)")

Output:

start index: 4
end index:   7
range:       4..<7
original:    This is a test
final:       !his is a test

Notice the indexes account for multiple code units. The flag (REGIONAL INDICATOR SYMBOL LETTERS ES) is 8 bytes and the (FACE WITH TEARS OF JOY) is 4 bytes. (In this particular case it turns out that the number of bytes is the same for UTF-8, UTF-16 and UTF-32 representations.)

Wrapping it in a func:

func replaceString(#string:String, #with:String, #start:Int, #length:Int) ->String {
    var startIndex = advance(original.startIndex, start) // Start at the second character
    var endIndex   = advance(startIndex, length) // point ahead two characters
    var range      = Range(start:startIndex, end:endIndex)
    var final = original.stringByReplacingCharactersInRange(range, withString: replacement)
    return final
}

var newString = replaceString(string:original, with:replacement, start:1, length:2)
println("newString:\(newString)")

Output:

newString: !his is a test

Android Animation Alpha

Setting alpha to 1 before starting the animation worked for me:

AlphaAnimation animation1 = new AlphaAnimation(0.2f, 1.0f);
animation1.setDuration(500);
iv.setAlpha(1f);
iv.startAnimation(animation1);

At least on my tests, there's no flickering because of setting alpha before starting the animation. It just works fine.

How do I convert from BLOB to TEXT in MySQL?

If you are using MYSQL-WORKBENCH, then you can select blob column normally and right click on column and click open value in editor. refer screenshot:

screenshot

Find non-ASCII characters in varchar columns using SQL Server

To find which field has invalid characters:

SELECT * FROM Staging.APARMRE1 FOR XML AUTO, TYPE

You can test it with this query:

SELECT top 1 'char 31: '+char(31)+' (hex 0x1F)' field
from sysobjects
FOR XML AUTO, TYPE

The result will be:

Msg 6841, Level 16, State 1, Line 3 FOR XML could not serialize the data for node 'field' because it contains a character (0x001F) which is not allowed in XML. To retrieve this data using FOR XML, convert it to binary, varbinary or image data type and use the BINARY BASE64 directive.

It is very useful when you write xml files and get error of invalid characters when validate it.

linux/videodev.h : no such file or directory - OpenCV on ubuntu 11.04

v4l support has been dropped in recent kernel versions (including the one shipped with Ubuntu 11.04).

EDIT: Your question is connected to a recent message that was sent to the OpenCV users group, which has instructions to compile OpenCV 2.2 in Ubuntu 11.04. Your approach is not ideal.

How can I color dots in a xy scatterplot according to column value?

If you code your x axis text categories, list them in a single column, then in adjacent columns list plot points for respective variables against relevant text category code and just leave blank cells against non-relevant text category code, you can scatter plot and get the displayed result. Any questions let me know. enter image description here

Error: Cannot pull with rebase: You have unstaged changes

Follow the below steps

From feature/branch (enter the below command)

git checkout master

git pull

git checkout feature/branchname

git merge master

Fixing "Lock wait timeout exceeded; try restarting transaction" for a 'stuck" Mysql table?

Goto processes in mysql.

So can see there is task still working.

Kill the particular process or wait until process complete.

Emulator: ERROR: x86 emulation currently requires hardware acceleration

In the android sdk manager it says that it has hardware accelerator already installed but I think it is only downloaded not installed.I just manually installed the intelhaxm-android.exe and it worked for me.

plus I had docker installed , there were some answers to disable Hyper-V features, therefore I did that too, but not sure whether it helped or not.

Is there a way to use max-width and height for a background image?

Unfortunately there's no min (or max)-background-size in CSS you can only use background-size. However if you are seeking a responsive background image you can use Vmin and Vmaxunits for the background-size property to achieve something similar.

example:

#one {
    background:url('../img/blahblah.jpg') no-repeat;
    background-size:10vmin 100%;
}

that will set the height to 10% of the whichever smaller viewport you have whether vertical or horizontal, and will set the width to 100%.

Read more about css units here: https://www.w3schools.com/cssref/css_units.asp

100% width Twitter Bootstrap 3 template

For Bootstrap 3, you would need to use a custom wrapper and set its width to 100%.

.container-full {
  margin: 0 auto;
  width: 100%;
}

Here is a working example on Bootply

If you prefer not to add a custom class, you can acheive a very wide layout (not 100%) by wrapping everything inside a col-lg-12 (wide layout demo)

Update for Bootstrap 3.1

The container-fluid class has returned in Bootstrap 3.1, so this can be used to create a full width layout (no additional CSS required)..

Bootstrap 3.1 demo

Navigation drawer: How do I set the selected item at startup?

Following code will only make menu item selected:

navigationView.setCheckedItem(id);

To select and open the menu item, add following code after the above line.

onNavigationItemSelected(navigationView.getMenu().getItem(0));

How to retrieve all keys (or values) from a std::map and put them into a vector?

(I'm always wondering why std::map does not include a member function for us to do so.)

Because it can't do it any better than you can do it. If a method's implementation will be no superior to a free function's implementation then in general you should not write a method; you should write a free function.

It's also not immediately clear why it's useful anyway.

Make footer stick to bottom of page correctly

do it using jQuery put inside code on the <head></head> tag

<script type="text/javascript">
$(document).ready(function() { 
    var docHeight = $(window).height();
    var footerHeight = $('#footer').height();
    var footerTop = $('#footer').position().top + footerHeight;  
    if (footerTop < docHeight) {
        $('#footer').css('margin-top', 10 + (docHeight - footerTop) + 'px');
    }
});
</script>

How to replace local branch with remote branch entirely in Git?

The ugly but simpler way: delete your local folder, and clone the remote repository again.

How to check if a char is equal to an empty space?

Character.isSpaceChar(c) || Character.isWhitespace(c) worked for me.

Method List in Visual Studio Code

Invoke Code's Go to symbol command:

  • macOS: cmd+shift+o (the letter o, not zero)

  • Windows/Linux: ctrl+shift+o

Typing a colon (:) after invoking Go to symbol will group symbols by type (classes, interfaces, methods, properties, variables). Then just scroll to the methods section.

Allowed memory size of 536870912 bytes exhausted in Laravel

for xampp it there is in xampp\php\php.ini now mine new option in it looks as :

;Maximum amount of memory a script may consume
;http://php.net/memory-limit
memory_limit=2048M
;memory_limit=512M

Creating a UITableView Programmatically

- (void)viewDidLoad {
     [super viewDidLoad];
     arr=[[NSArray alloc]initWithObjects:@"ABC",@"XYZ", nil];
     tableview = [[UITableView alloc]initWithFrame:tableFrame style:UITableViewStylePlain];   
     tableview.delegate = self;
     tableview.dataSource = self;
     [self.view addSubview:tableview];
}

- (NSInteger)tableView:(UITableView *)tableView numberOfRowsInSection:(NSInteger)section
{
    return arr.count;
}

- (UITableViewCell *)tableView:(UITableView *)tableView cellForRowAtIndexPath:(NSIndexPath *)indexPath
{
    UITableViewCell *cell = [tableView dequeueReusableCellWithIdentifier:@"MyCell"];

    if(cell == nil)
    {
        cell = [[UITableViewCell alloc]initWithStyle:UITableViewCellStyleDefault reuseIdentifier:@"MyCell"];
    }

    cell.textLabel.text=[arr objectAtIndex:indexPath.row];

    return cell;
}

Removing certain characters from a string in R

try: gsub('\\$', '', '$5.00$')

Remove last characters from a string in C#. An elegant way?

Use:

public static class StringExtensions
{
    /// <summary>
    /// Cut End. "12".SubstringFromEnd(1) -> "1"
    /// </summary>
    public static string SubstringFromEnd(this string value, int startindex)
    {
        if (string.IsNullOrEmpty(value)) return value;
        return value.Substring(0, value.Length - startindex);
    }
}

I prefer an extension method here for two reasons:

  1. I can chain it with Substring. Example: f1.Substring(directorypathLength).SubstringFromEnd(1)
  2. Speed.

Remove blank attributes from an Object in Javascript

JSON.stringify removes the undefined keys.

removeUndefined = function(json){
  return JSON.parse(JSON.stringify(json))
}

What USB driver should we use for the Nexus 5?

I am on Windows 8.1, and I tried everything from the other answers and nothing worked. Finally, I decided to try "Pick from a list of drivers" and found "LGE Mobile Sooner Single ADB Interface" under "ADB Interface". ADB.exe was finally able to find the Nexus 5 and sideload 4.4.1.

I hope this helps.

SwiftUI - How do I change the background color of a View?

I like to declare a modifier for changing the background color of a view.

extension View {
  func background(with color: Color) -> some View {
    background(GeometryReader { geometry in
      Rectangle().path(in: geometry.frame(in: .local)).foregroundColor(color)
    })
  }
}

Then I use the modifier by passing in a color to a view.

struct Content: View {

  var body: some View {
    Text("Foreground Label").foregroundColor(.green).background(with: .black)
  }

}

enter image description here

How do I add PHP code/file to HTML(.html) files?

You can't run PHP in .html files because the server does not recognize that as a valid PHP extension unless you tell it to. To do this you need to create a .htaccess file in your root web directory and add this line to it:

AddType application/x-httpd-php .htm .html

This will tell Apache to process files with a .htm or .html file extension as PHP files.

Using psql to connect to PostgreSQL in SSL mode

Well, you cloud provide all the information with following command in CLI, if connection requires in SSL mode:

psql "sslmode=verify-ca sslrootcert=server-ca.pem sslcert=client-cert.pem sslkey=client-key.pem hostaddr=your_host port=5432 user=your_user dbname=your_db" 

Set up Python 3 build system with Sublime Text 3

first you need to find your python.exe location, to find location run this python script:

import sys
print(sys.executable)

Then you can create your custom python build:

{
"cmd": ["C:\\Users\\Sashi\\AppData\\Local\\Programs\\Python\\Python39\\python.exe", "-u", "$file"],
"file_regex": "^[ ]File \"(...?)\", line ([0-9]*)",
"selector": "source.python"}

You can change the location, In my case it is C:\Users\Sashi\AppData\Local\Programs\Python\Python39\python.exe

Then save your new build. Don't change the file extension while saving.

How can I add a key/value pair to a JavaScript object?

In case you have multiple anonymous Object literals inside an Object and want to add another Object containing key/value pairs, do this:

Firebug' the Object:

console.log(Comicbook);

returns:

[Object { name="Spiderman", value="11"}, Object { name="Marsipulami", value="18"}, Object { name="Garfield", value="2"}]

Code:

if (typeof Comicbook[3]=='undefined') {
    private_formArray[3] = new Object();
    private_formArray[3]["name"] = "Peanuts";
    private_formArray[3]["value"] = "12";
}

will add Object {name="Peanuts", value="12"} to the Comicbook Object

MySQL: Curdate() vs Now()

For questions like this, it is always worth taking a look in the manual first. Date and time functions in the mySQL manual

CURDATE() returns the DATE part of the current time. Manual on CURDATE()

NOW() returns the date and time portions as a timestamp in various formats, depending on how it was requested. Manual on NOW().

Can I change the viewport meta tag in mobile safari on the fly?

I realize this is a little old, but, yes it can be done. Some javascript to get you started:

viewport = document.querySelector("meta[name=viewport]");
viewport.setAttribute('content', 'width=device-width, initial-scale=1.0, maximum-scale=1.0, user-scalable=0');

Just change the parts you need and Mobile Safari will respect the new settings.

Update:

If you don't already have the meta viewport tag in the source, you can append it directly with something like this:

var metaTag=document.createElement('meta');
metaTag.name = "viewport"
metaTag.content = "width=device-width, initial-scale=1.0, maximum-scale=1.0, user-scalable=0"
document.getElementsByTagName('head')[0].appendChild(metaTag);

Or if you're using jQuery:

$('head').append('<meta name="viewport" content="width=device-width, initial-scale=1.0, maximum-scale=1.0, user-scalable=0">');

Place a button right aligned

If the button is the only element on the block:

_x000D_
_x000D_
.border {_x000D_
  border: 2px blue dashed;_x000D_
}_x000D_
_x000D_
.mr-0 {_x000D_
  margin-right: 0;_x000D_
}_x000D_
.ml-auto {_x000D_
  margin-left:auto;_x000D_
}_x000D_
.d-block {_x000D_
  display:block;_x000D_
}
_x000D_
<p class="border">_x000D_
  <input type="button" class="d-block mr-0 ml-auto" value="The Button">_x000D_
</p>
_x000D_
_x000D_
_x000D_

If there are other elements on the block:

_x000D_
_x000D_
.border {_x000D_
  border: 2px indigo dashed;_x000D_
}_x000D_
_x000D_
.d-table {_x000D_
  display:table;_x000D_
}_x000D_
_x000D_
.d-table-cell {_x000D_
  display:table-cell;_x000D_
}_x000D_
_x000D_
.w-100 {_x000D_
  width: 100%;_x000D_
}_x000D_
_x000D_
.tar {_x000D_
  text-align: right;_x000D_
}
_x000D_
<div class="border d-table w-100">_x000D_
  <p class="d-table-cell">The paragraph.....lorem ipsum...etc.</p>_x000D_
  <div class="d-table-cell tar">_x000D_
    <button >The Button</button>_x000D_
  </div>_x000D_
</div>
_x000D_
_x000D_
_x000D_

With flex-box:

_x000D_
_x000D_
.flex-box {_x000D_
  display:flex;_x000D_
  justify-content:space-between;_x000D_
  outline: 2px dashed blue;_x000D_
}_x000D_
_x000D_
.flex-box-2 {_x000D_
  display:flex;_x000D_
  justify-content: flex-end;_x000D_
  outline: 2px deeppink dashed;_x000D_
}
_x000D_
<h1>Button with Text</h1>_x000D_
<div class="flex-box">_x000D_
<p>Once upon a time in a ...</p>_x000D_
<button>Read More...</button>_x000D_
</div>_x000D_
_x000D_
<h1>Only Button</h1>_x000D_
<div class="flex-box-2">_x000D_
  <button>The Button</button>_x000D_
</div>_x000D_
_x000D_
<h1>Multiple Buttons</h1>_x000D_
<div class="flex-box-2">_x000D_
  <button>Button 1</button>_x000D_
  <button>Button 2</button>_x000D_
</div>
_x000D_
_x000D_
_x000D_

Good Luck...

How to convert TimeStamp to Date in Java?

// timestamp to Date
long timestamp = 5607059900000; //Example -> in ms
Date d = new Date(timestamp );

// Date to timestamp
long timestamp = d.getTime();

//If you want the current timestamp :
Calendar c = Calendar.getInstance();
long timestamp = c.getTimeInMillis();

Where do I get servlet-api.jar from?

Make sure that you're using the same Servlet API specification that your Web container supports. Refer to this chart if you're using Tomcat: http://tomcat.apache.org/whichversion.html

The Web container that you use will definitely have the API jars you require.

Tomcat 6 for example has it in apache-tomcat-6.0.26/lib/servlet-api.jar

How to get PHP $_GET array?

Put all the ids into a variable called $ids and separate them with a ",":

$ids = "1,2,3,4,5,6";

Pass them like so:

$url = "?ids={$ids}";

Process them:

$ids = explode(",", $_GET['ids']);

How do I set up Android Studio to work completely offline?

OK guys I finally overcame this problem. Here is the solution:

  1. Download gradle-1.6-bin.zip for offline use.

  2. Paste it in the C:\Users\username\.gradle directory.

  3. Open Android Studio and click on the "Create New Project" option and you will not get this error any more while offline.

    You might get some other errors like this:

    Don't worry, just ignore it. Your project has been created.

  4. So now click on "Import Project" and go to the path C:\Users\username\AndroidStudioProjects and open your project and you are done.

Calling remove in foreach loop in Java

Make sure this is not code smell. Is it possible to reverse the logic and be 'inclusive' rather than 'exclusive'?

List<String> names = ....
List<String> reducedNames = ....
for (String name : names) {
   // Do something
   if (conditionToIncludeMet)
       reducedNames.add(name);
}
return reducedNames;

The situation that led me to this page involved old code that looped through a List using indecies to remove elements from the List. I wanted to refactor it to use the foreach style.

It looped through an entire list of elements to verify which ones the user had permission to access, and removed the ones that didn't have permission from the list.

List<Service> services = ...
for (int i=0; i<services.size(); i++) {
    if (!isServicePermitted(user, services.get(i)))
         services.remove(i);
}

To reverse this and not use the remove:

List<Service> services = ...
List<Service> permittedServices = ...
for (Service service:services) {
    if (isServicePermitted(user, service))
         permittedServices.add(service);
}
return permittedServices;

When would "remove" be preferred? One consideration is if gien a large list or expensive "add", combined with only a few removed compared to the list size. It might be more efficient to only do a few removes rather than a great many adds. But in my case the situation did not merit such an optimization.

Draw Circle using css alone

border radius is good option, if struggling with old IE versions then try HTML codes

&#8226;

and use css to change color. Output:

How to preview a part of a large pandas DataFrame, in iPython notebook?

df.head(5) # will print out the first 5 rows
df.tail(5) # will print out the 5 last rows

Linux command (like cat) to read a specified quantity of characters

Here's a simple script that wraps up using the dd approach mentioned here:

extract_chars.sh

#!/usr/bin/env bash

function show_help()
{
  IT="
extracts characters X to Y from stdin or FILE
usage: X Y {FILE}

e.g. 

2 10 /tmp/it     => extract chars 2-10 from /tmp/it
EOF
  "
  echo "$IT"
  exit
}

if [ "$1" == "help" ]
then
  show_help
fi
if [ -z "$1" ]
then
  show_help
fi

FROM=$1
TO=$2
COUNT=`expr $TO - $FROM + 1`

if [ -z "$3" ]
then
  dd skip=$FROM count=$COUNT bs=1 2>/dev/null
else
  dd skip=$FROM count=$COUNT bs=1 if=$3 2>/dev/null 
fi

Adding a guideline to the editor in Visual Studio

For those who use Visual Assist, vertical guidelines can be enabled from Display section in Visual Assist's options:

enter image description here

javascript Unable to get property 'value' of undefined or null reference

you have many HTML and java script mistakes includes: tag, using non UTF-8 encoding for form submission, no need,... You must use document.forms.FORMNAME or document.forms[0] for first appear form in page Corrected:

_x000D_
_x000D_
 function validate_frm_new_user_request()_x000D_
{_x000D_
    alert('test');_x000D_
    var valid = true;_x000D_
_x000D_
    if ( document.forms.frm_new_user_request.u_userid.value == "" )_x000D_
    {_x000D_
        alert ( "Please enter your valid ISID Information." );_x000D_
        document.forms.frm_new_user_request.u_userid.focus();_x000D_
        valid = false;_x000D_
  console.log("FALSE::Empty Value ");_x000D_
    }_x000D_
return valid;_x000D_
}
_x000D_
<html lang="en" xml:lang="en" xmlns="http://www.w3.org/1999/xhtml">_x000D_
<head>_x000D_
    <title></title>_x000D_
    <meta content="text/html;charset=UTF-8" http-equiv="content-type" />_x000D_
_x000D_
_x000D_
</head>_x000D_
<body>_x000D_
<form method="post" action="" name="frm_new_user_request" id="frm_new_user_request" onsubmit="return validate_frm_new_user_request();">_x000D_
<center>_x000D_
<table>_x000D_
_x000D_
        <tr align="left">_x000D_
            <td><Label>ISID<em>*:</Label><input maxlength="15" id="u_userid" name="u_userid" size="20" type="text"/></td>_x000D_
            </tr>_x000D_
_x000D_
<tr>_x000D_
            <td align="center" colspan="4">_x000D_
                <input type="image" src="btn.png" border="0" ALT="Create New Request">_x000D_
_x000D_
                </td>_x000D_
        </tr>_x000D_
    </table>_x000D_
</form>_x000D_
</body>_x000D_
</html>
_x000D_
_x000D_
_x000D_

Pod install is staying on "Setting up CocoaPods Master repo"

None of the solutions above worked for me, I had to uninstall coacoapods, then installed a specific version before everything worked for me

sudo gem uninstall cocoapods

then

sudo gem install cocoapods -v 1.7.5

now even verbose shows progress

$ pod setup --verbose

Setting up CocoaPods master repo

Cloning spec repo `master` from `https://github.com/CocoaPods/Specs.git` (branch `master`)
  $ /usr/bin/git clone https://github.com/CocoaPods/Specs.git --progress -- master
  Cloning into 'master'...
  remote: Enumerating objects: 295, done.        
  remote: Counting objects: 100% (295/295), done.        
  remote: Compressing objects: 100% (283/283), done.        
  Receiving objects:  20% (744493/3722462), 132.93 MiB | 567.00 KiB/s   

jQuery UI Dialog Box - does not open after being closed

 <button onClick="abrirOpen()">Open Dialog</button>

<script type="text/javascript">
var $dialogo = $("<div></div>").html("Aqui tu contenido(here your content)").dialog({
       title: "Dialogo de UI",
       autoOpen: false,
       close: function(ev, ui){
               $(this).dialog("destroy");
       }
 function abrirOpen(){
       $dialogo.dialog("open");
 }
});

//**Esto funciona para mi... (this works for me)**
</script>

Using only CSS, show div on hover over <a>

_x000D_
_x000D_
.showme {_x000D_
  display: none;_x000D_
}_x000D_
_x000D_
.showhim:hover .showme {_x000D_
  display: block;_x000D_
}
_x000D_
<div class="showhim">HOVER ME_x000D_
  <div class="showme">hai</div>_x000D_
</div>
_x000D_
_x000D_
_x000D_

jsfiddle

Since this answer is popular I think a small explanation is needed. Using this method when you hover on the internal element, it wont disappear. Because the .showme is inside .showhim it will not disappear when you move your mouse between the two lines of text (or whatever it is).

These are example of quirqs you need to take care of when implementing such behavior.

It all depends what you need this for. This method is better for a menu style scenario, while Yi Jiang's is better for tooltips.

How to use setInterval and clearInterval?

setInterval sets up a recurring timer. It returns a handle that you can pass into clearInterval to stop it from firing:

var handle = setInterval(drawAll, 20);

// When you want to cancel it:
clearInterval(handle);
handle = 0; // I just do this so I know I've cleared the interval

On browsers, the handle is guaranteed to be a number that isn't equal to 0; therefore, 0 makes a handy flag value for "no timer set". (Other platforms may return other values; NodeJS's timer functions return an object, for instance.)

To schedule a function to only fire once, use setTimeout instead. It won't keep firing. (It also returns a handle you can use to cancel it via clearTimeout before it fires that one time if appropriate.)

setTimeout(drawAll, 20);

Angular 2 Sibling Component Communication

This is not what you exactly want but for sure will help you out

I'm surprised there's not more information out there about component communication <=> consider this tutorial by angualr2

For sibling components communication, I'd suggest to go with sharedService. There are also other options available though.

import {Component,bind} from 'angular2/core';
import {bootstrap} from 'angular2/platform/browser';
import {HTTP_PROVIDERS} from 'angular2/http';
import {NameService} from 'src/nameService';


import {TheContent} from 'src/content';
import {Navbar} from 'src/nav';


@Component({
  selector: 'app',
  directives: [TheContent,Navbar],
  providers: [NameService],
  template: '<navbar></navbar><thecontent></thecontent>'
})


export class App {
  constructor() {
    console.log('App started');
  }
}

bootstrap(App,[]);

Please refer to link at top for more code.

Edit: This is a very small demo. You have already mention that you have already tried with sharedService. So please consider this tutorial by angualr2 for more information.

How can I store HashMap<String, ArrayList<String>> inside a list?

Always try to use interface reference in Collection, this adds more flexibility.
What is the problem with the below code?

List<Map<String,List<String>>> list = new ArrayList<Map<String,List<String>>>();//This is the final list you need
Map<String, List<String>> map1 = new HashMap<String, List<String>>();//This is one instance of the  map you want to store in the above list.
List<String> arraylist1 = new ArrayList<String>();
arraylist1.add("Text1");//And so on..
map1.put("key1",arraylist1);
//And so on...
list.add(map1);//In this way you can add.

You can easily do it like the above.

How to annotate MYSQL autoincrement field with JPA annotations

Using MySQL, only this approach was working for me:

@Id @GeneratedValue(strategy=GenerationType.IDENTITY)
private Long id;

The other 2 approaches stated by Pascal in his answer were not working for me.

How to use GROUP BY to concatenate strings in SQL Server?

Eight years later... Microsoft SQL Server vNext Database Engine has finally enhanced Transact-SQL to directly support grouped string concatenation. The Community Technical Preview version 1.0 added the STRING_AGG function and CTP 1.1 added the WITHIN GROUP clause for the STRING_AGG function.

Reference: https://msdn.microsoft.com/en-us/library/mt775028.aspx

How do I tell CMake to link in a static library in the source directory?

CMake favours passing the full path to link libraries, so assuming libbingitup.a is in ${CMAKE_SOURCE_DIR}, doing the following should succeed:

add_executable(main main.cpp)
target_link_libraries(main ${CMAKE_SOURCE_DIR}/libbingitup.a)

How to remove the character at a given index from a string in C?

The following will extends the problem a bit by removing from the first string argument any character that occurs in the second string argument.

/*
 * delete one character from a string
 */
static void
_strdelchr( char *s, size_t i, size_t *a, size_t *b)
{
  size_t        j;

  if( *a == *b)
    *a = i - 1;
  else
    for( j = *b + 1; j < i; j++)
      s[++(*a)] = s[j];
  *b = i;
}

/*
 * delete all occurrences of characters in search from s
 * returns nr. of deleted characters
 */
size_t
strdelstr( char *s, const char *search)
{ 
  size_t        l               = strlen(s);
  size_t        n               = strlen(search);
  size_t        i;
  size_t        a               = 0;
  size_t        b               = 0;

  for( i = 0; i < l; i++)
    if( memchr( search, s[i], n))
      _strdelchr( s, i, &a, &b);
  _strdelchr( s, l, &a, &b);
  s[++a] = '\0';
  return l - a;
}

jquery - is not a function error

In my case, the same error had a much easier fix. Basically my function was in a .js file that was not included in the current aspx that was showing. All I needed was the include line.

What is aria-label and how should I use it?

The title attribute displays a tooltip when the mouse is hovering the element. While this is a great addition, it doesn't help people who cannot use the mouse (due to mobility disabilities) or people who can't see this tooltip (e.g.: people with visual disabilities or people who use a screen reader).

As such, the mindful approach here would be to serve all users. I would add both title and aria-label attributes (serving different types of users and different types of usage of the web).

Here's a good article that explains aria-label in depth

How to see my Eclipse version?

Open .eclipseproduct in the product installation folder. Or open Configuration\config.ini and check property eclipse.buildId if exist.

What does a just-in-time (JIT) compiler do?

You have code that is compliled into some IL (intermediate language). When you run your program, the computer doesn't understand this code. It only understands native code. So the JIT compiler compiles your IL into native code on the fly. It does this at the method level.

JQuery ajax call default timeout value

The XMLHttpRequest.timeout property represents a number of milliseconds a request can take before automatically being terminated. The default value is 0, which means there is no timeout. An important note the timeout shouldn't be used for synchronous XMLHttpRequests requests, used in a document environment or it will throw an InvalidAccessError exception. You may not use a timeout for synchronous requests with an owning window.

IE10 and 11 do not support synchronous requests, with support being phased out in other browsers too. This is due to detrimental effects resulting from making them.

More info can be found here.

How can I check if a command exists in a shell script?

A function I have in an install script made for exactly this

function assertInstalled() {
    for var in "$@"; do
        if ! which $var &> /dev/null; then
            echo "Install $var!"
            exit 1
        fi
    done
}

example call:

assertInstalled zsh vim wget python pip git cmake fc-cache

Conditional formatting based on another cell's value

change the background color of cell B5 based on the value of another cell - C5. If C5 is greater than 80% then the background color is green but if it's below, it will be amber/red.

There is no mention that B5 contains any value so assuming 80% is .8 formatted as percentage without decimals and blank counts as "below":

Select B5, colour "amber/red" with standard fill then Format - Conditional formatting..., Custom formula is and:

=C5>0.8

with green fill and Done.

CF rule example

What is difference between arm64 and armhf?

armhf stands for "arm hard float", and is the name given to a debian port for arm processors (armv7+) that have hardware floating point support.

On the beaglebone black, for example:

:~$ dpkg --print-architecture
armhf

Although other commands (such as uname -a or arch) will just show armv7l

:~$ cat /proc/cpuinfo 
processor       : 0
model name      : ARMv7 Processor rev 2 (v7l)
BogoMIPS        : 995.32
Features        : half thumb fastmult vfp edsp thumbee neon vfpv3 tls
...

The vfpv3 listed under Features is what refers to the floating point support.

Incidentally, armhf, if your processor supports it, basically supersedes Raspbian, which if I understand correctly was mainly a rebuild of armhf with work arounds to deal with the lack of floating point support on the original raspberry pi's. Nowdays, of course, there's a whole ecosystem build up around Raspbian, so they're probably not going to abandon it. However, this is partly why the beaglebone runs straight debian, and that's ok even if you're used to Raspbian, unless you want some of the special included non-free software such as Mathematica.

In python, how do I cast a class object to a dict

I think this will work for you.

class A(object):
    def __init__(self, a, b, c, sum, version='old'):
        self.a = a
        self.b = b
        self.c = c
        self.sum = 6
        self.version = version

    def __int__(self):
        return self.sum + 9000

    def __iter__(self):
        return self.__dict__.iteritems()

a = A(1,2,3,4,5)
print dict(a)

Output

{'a': 1, 'c': 3, 'b': 2, 'sum': 6, 'version': 5}

How to pass parameters to a modal?

You should really use Angular UI for that needs. Check it out: Angular UI Dialog

In a nutshell, with Angular UI dialog, you can pass variable from a controller to the dialog controller using resolve. Here's your "from" controller:

var d = $dialog.dialog({
  backdrop: true,
  keyboard: true,
  backdropClick: true,
  templateUrl:  '<url_of_your_template>',
  controller: 'MyDialogCtrl',
  // Interesting stuff here.
  resolve: {
    username: 'foo'
  }
});

d.open();

And in your dialog controller:

angular.module('mymodule')
  .controller('MyDialogCtrl', function ($scope, username) {
  // Here, username is 'foo'
  $scope.username = username;
}

EDIT: Since the new version of the ui-dialog, the resolve entry becomes:

resolve: { username: function () { return 'foo'; } }

Pip - Fatal error in launcher: Unable to create process using '"'

This worked for me under Windows 10 x64:

Ensure that the Python directories are in the path, e.g.:

# Edit Environment variables so that variable "path" points to the new location.
# Insert these at the start of the list (or delete other Python directories), as Windows takes the first match it finds.
# Run the program "Edit the System Environment Variables".
# Or see Control Panel under "System Properties".
S:\Research\bin\Python375\Scripts\
S:\Research\bin\Python375\

Then:

python -m pip install --upgrade --force-reinstall pip

In my particular case, the error was caused by shifting the Python directory to a new location.

Xpath: select div that contains class AND whose specific child element contains text

To find a div of a certain class that contains a span at any depth containing certain text, try:

//div[contains(@class, 'measure-tab') and contains(.//span, 'someText')]

That said, this solution looks extremely fragile. If the table happens to contain a span with the text you're looking for, the div containing the table will be matched, too. I'd suggest to find a more robust way of filtering the elements. For example by using IDs or top-level document structure.

Cannot resolve method 'getSupportFragmentManager ( )' inside Fragment

For my case, I made sure that Fragment class is imported from

android.support.v4.app.Fragment

Not from

android.app.Fragment

Then I have used

getActivity().getSupportFragmentManager();

And now its working. If I use activity instance which I got in onAttach() is not working also.

How to check variable type at runtime in Go language

The answer by @Darius is the most idiomatic (and probably more performant) method. One limitation is that the type you are checking has to be of type interface{}. If you use a concrete type it will fail.

An alternative way to determine the type of something at run-time, including concrete types, is to use the Go reflect package. Chaining TypeOf(x).Kind() together you can get a reflect.Kind value which is a uint type: http://golang.org/pkg/reflect/#Kind

You can then do checks for types outside of a switch block, like so:

import (
    "fmt"
    "reflect"
)

// ....

x := 42
y := float32(43.3)
z := "hello"

xt := reflect.TypeOf(x).Kind()
yt := reflect.TypeOf(y).Kind()
zt := reflect.TypeOf(z).Kind()

fmt.Printf("%T: %s\n", xt, xt)
fmt.Printf("%T: %s\n", yt, yt)
fmt.Printf("%T: %s\n", zt, zt)

if xt == reflect.Int {
    println(">> x is int")
}
if yt == reflect.Float32 {
    println(">> y is float32")
}
if zt == reflect.String {
    println(">> z is string")
}

Which prints outs:

reflect.Kind: int
reflect.Kind: float32
reflect.Kind: string
>> x is int
>> y is float32
>> z is string

Again, this is probably not the preferred way to do it, but it's good to know alternative options.

how to remove new lines and returns from php string?

$no_newlines = str_replace("\r", '', str_replace("\n", '', $str_with_newlines));

How to extract the file name from URI returned from Intent.ACTION_GET_CONTENT?

My version of the answer is actually very similar to the @Stefan Haustein. I found the answer on Android Developer page Retrieving File Information; the information here is even more condensed on this specific topic than on Storage Access Framework guide site. In the result from the query the column index containing file name is OpenableColumns.DISPLAY_NAME. None of other answers/solutions for column indexes worked for me. Below is the sample function:

 /**
 * @param uri uri of file.
 * @param contentResolver access to server app.
 * @return the name of the file.
 */
def extractFileName(uri: Uri, contentResolver: ContentResolver): Option[String] = {

    var fileName: Option[String] = None
    if (uri.getScheme.equals("file")) {

        fileName = Option(uri.getLastPathSegment)
    } else if (uri.getScheme.equals("content")) {

        var cursor: Cursor = null
        try {

            // Query the server app to get the file's display name and size.
            cursor = contentResolver.query(uri, null, null, null, null)

            // Get the column indexes of the data in the Cursor,
            // move to the first row in the Cursor, get the data.
            if (cursor != null && cursor.moveToFirst()) {

                val nameIndex = cursor.getColumnIndex(OpenableColumns.DISPLAY_NAME)
                fileName = Option(cursor.getString(nameIndex))
            }

        } finally {

            if (cursor != null) {
                cursor.close()
            }

        }

    }

    fileName
}

How to get unique values in an array

Using EcmaScript 2016 you can simply do it like this.

 var arr = ["a", "a", "b"];
 var uniqueArray = Array.from(new Set(arr)); // Unique Array ['a', 'b'];

Sets are always unique, and using Array.from() you can convert a Set to an array. For reference have a look at the documentations.

https://developer.mozilla.org/en-US/docs/Web/JavaScript/Reference/Global_Objects/Array/from https://developer.mozilla.org/en-US/docs/Web/JavaScript/Reference/Global_Objects/Set

Creating a border like this using :before And :after Pseudo-Elements In CSS?

See the following snippet, is this what you want?

_x000D_
_x000D_
body {
    background: silver;
    padding: 0 10px;
}

#content:after {
    height: 10px;
    display: block;
    width: 100px;
    background: #808080;
    border-right: 1px white;
    content: '';
}

#footer:before {
    display: block;
    content: '';
    background: silver;
    height: 10px;
    margin-top: -20px;
    margin-left: 101px;
}

#content {
    background: white;
}


#footer {
    padding-top: 10px;
    background: #404040;
}

p {
    padding: 100px;
    text-align: center;
}

#footer p {
    color: white;
}
_x000D_
<body>
    <div id="content"><p>#content</p></div>
    <div id="footer"><p>#footer</p></div>
</body>
_x000D_
_x000D_
_x000D_

JSFiddle

Comprehensive methods of viewing memory usage on Solaris

"top" is usually available on Solaris.

If not then revert to "vmstat" which is available on most UNIX system.

It should look something like this (from an AIX box)

vmstat

System configuration: lcpu=4 mem=12288MB ent=2.00

kthr    memory              page              faults              cpu
----- ----------- ------------------------ ------------ -----------------------
 r  b   avm   fre  re  pi  po  fr   sr  cy  in   sy  cs us sy id wa    pc    ec
 2  1 1614644 585722   0   0   1  22  104   0 808 29047 2767 12  8 77  3  0.45  22.3

the colums "avm" and "fre" tell you the total memory and free memery.

a "man vmstat" should get you the gory details.

Swift Alamofire: How to get the HTTP response status code

Or use pattern matching

if let error = response.result.error as? AFError {
   if case .responseValidationFailed(.unacceptableStatusCode(let code)) = error {
       print(code)
   }
}

How to revert a "git rm -r ."?

undo git rm

git rm file             # delete file & update index
git checkout HEAD file  # restore file & index from HEAD

undo git rm -r

git rm -r dir          # delete tracked files in dir & update index
git checkout HEAD dir  # restore file & index from HEAD

undo git rm -rf

git rm -r dir          # delete tracked files & delete uncommitted changes
not possible           # `uncommitted changes` can not be restored.

Uncommitted changes includes not staged changes, staged changes but not committed.

How to get input text value from inside td

I'm having a hard time figuring out what exactly you're looking for here, so hope I'm not way off base.

I'm assuming what you mean is that when a keyup event occurs on the input with class "start" you want to get the values of all the inputs in neighbouring <td>s:

    $('.start').keyup(function() {
        var otherInputs = $(this).parents('td').siblings().find('input');

        for(var i = 0; i < otherInputs.length; i++) {
            alert($(otherInputs[i]).val());
        }
        return false;
    });

Check if a number is a perfect square

import math

def is_square(n):
    sqrt = math.sqrt(n)
    return (sqrt - int(sqrt)) == 0

A perfect square is a number that can be expressed as the product of two equal integers. math.sqrt(number) return a float. int(math.sqrt(number)) casts the outcome to int.

If the square root is an integer, like 3, for example, then math.sqrt(number) - int(math.sqrt(number)) will be 0, and the if statement will be False. If the square root was a real number like 3.2, then it will be True and print "it's not a perfect square".

It fails for a large non-square such as 152415789666209426002111556165263283035677490.

Insert the same fixed value into multiple rows

UPDATE `table` SET table_column='test';

How to force a checkbox and text on the same line?

Another way to do this solely with css:

input[type='checkbox'] {
  float: left;
  width: 20px;
}
input[type='checkbox'] + label {
  display: block;
  width: 30px;
}

Note that this forces each checkbox and its label onto a separate line, rather than only doing so only when there's overflow.

How to fill Matrix with zeros in OpenCV?

Because the last parameter is optional and also the data pointer should point somewhere appropriate:

//inline Mat::Mat(int _rows, int _cols, int _type, void* _data, size_t _step)
double mydata[1];
Mat m1 = Mat(1,1, CV_64F, mydata); 
m1.at<double>(0,0) = 0;

But better do it directly with this template-based constructor:

//inline Mat::Mat(int _rows, int _cols, int _type, const Scalar& _s)
Mat m1 = Mat(1,1, CV_64F, cvScalar(0.));

//or even
Mat m1 = Mat(1,1, CV_64F, double(0));

How to check "hasRole" in Java Code with Spring Security?

Strangely enough, I don't think there is a standard solution to this problem, as the spring-security access control is expression based, not java-based. you might check the source code for DefaultMethodSecurityExpressionHandler to see if you can re-use something they are doing there

Use space as a delimiter with cut command

cut -d ' ' -f 2

Where 2 is the field number of the space-delimited field you want.

Quick easy way to migrate SQLite3 to MySQL?

If you are using Python/Django it's pretty easy:

create two databases in settings.py (like here https://docs.djangoproject.com/en/1.11/topics/db/multi-db/)

then just do like this:

objlist = ModelObject.objects.using('sqlite').all()

for obj in objlist:
    obj.save(using='mysql')

Cross browser JavaScript (not jQuery...) scroll to top animation

Without JQuery code, Hope this will help you.

function TopscrollTo() {
if(window.scrollY!=0)
{
    setTimeout(function() {
       window.scrollTo(0,window.scrollY-30);
        TopscrollTo();
    }, 100);
   }
}

call this TopscrollTo() function on button click event or on any other element/event which you want.

Turning off eslint rule for a specific file

It's better to add "overrides" in your .eslintrc.js config file. For example if you wont to disable camelcase rule for all js files ending on Actions add this code after rules scope in .eslintrc.js.

"rules": {    
...........    
},
"overrides": [
 {
  "files": ["*Actions.js"],
     "rules": {
        "camelcase": "off"   
     }
 }
]

SQL Server database restore error: specified cast is not valid. (SqlManagerUI)

Could be because of restoring SQL Server 2012 version backup file into SQL Server 2008 R2 or even less.

Understanding events and event handlers in C#

That is actually the declaration for an event handler - a method that will get called when an event is fired. To create an event, you'd write something like this:

public class Foo
{
    public event EventHandler MyEvent;
}

And then you can subscribe to the event like this:

Foo foo = new Foo();
foo.MyEvent += new EventHandler(this.OnMyEvent);

With OnMyEvent() defined like this:

private void OnMyEvent(object sender, EventArgs e)
{
    MessageBox.Show("MyEvent fired!");
}

Whenever Foo fires off MyEvent, then your OnMyEvent handler will be called.

You don't always have to use an instance of EventArgs as the second parameter. If you want to include additional information, you can use a class derived from EventArgs (EventArgs is the base by convention). For example, if you look at some of the events defined on Control in WinForms, or FrameworkElement in WPF, you can see examples of events that pass additional information to the event handlers.

Automatic login script for a website on windows machine?

You can use Autohotkey, download it from: http://ahkscript.org/download/

After the installation, if you want to open Gmail website when you press Alt+g, you can do something like this:

!g::
Run www.gmail.com 
return

Further reference: Hotkeys (Mouse, Joystick and Keyboard Shortcuts)

How to get a Fragment to remove itself, i.e. its equivalent of finish()?

OnCreate:

//Add comment fragment
            container = FindViewById<FrameLayout>(Resource.Id.frmAttachPicture);
            mPictureFragment = new fmtAttachPicture();

            var trans = SupportFragmentManager.BeginTransaction();
            trans.Add(container.Id, mPictureFragment, "fmtPicture");
            trans.Show(mPictureFragment); trans.Commit();

This is how I hide the fragment in click event 1

//Close fragment
    var trans = SupportFragmentManager.BeginTransaction();
    trans.Hide(mPictureFragment);
    trans.AddToBackStack(null);
    trans.Commit();

Then Shows it back int event 2

var trans = SupportFragmentManager.BeginTransaction();
            trans.Show(mPictureFragment); trans.Commit();

The type java.lang.CharSequence cannot be resolved in package declaration

Make your Project and Workspace to point to JDK7 which will resolve the issue. https://developers.google.com/eclipse/docs/jdk_compliance has given ways to modify Compliance and Facet level changes.

START_STICKY and START_NOT_STICKY

KISS answer

Difference:

START_STICKY

the system will try to re-create your service after it is killed

START_NOT_STICKY

the system will not try to re-create your service after it is killed


Standard example:

@Override
public int onStartCommand(Intent intent, int flags, int startId) {
    return START_STICKY;
}

Using Gulp to Concatenate and Uglify files

My gulp file produces a final compiled-bundle-min.js, hope this helps someone.

enter image description here

//Gulpfile.js

var gulp = require("gulp");
var watch = require("gulp-watch");

var concat = require("gulp-concat");
var rename = require("gulp-rename");
var uglify = require("gulp-uglify");
var del = require("del");
var minifyCSS = require("gulp-minify-css");
var copy = require("gulp-copy");
var bower = require("gulp-bower");
var sourcemaps = require("gulp-sourcemaps");

var path = {
    src: "bower_components/",
    lib: "lib/"
}

var config = {
    jquerysrc: [
        path.src + "jquery/dist/jquery.js",
        path.src + "jquery-validation/dist/jquery.validate.js",
        path.src + "jquery-validation/dist/jquery.validate.unobtrusive.js"
    ],
    jquerybundle: path.lib + "jquery-bundle.js",
    ngsrc: [
        path.src + "angular/angular.js",
         path.src + "angular-route/angular-route.js",
         path.src + "angular-resource/angular-resource.js"
    ],
    ngbundle: path.lib + "ng-bundle.js",

    //JavaScript files that will be combined into a Bootstrap bundle
    bootstrapsrc: [
        path.src + "bootstrap/dist/js/bootstrap.js"
    ],
    bootstrapbundle: path.lib + "bootstrap-bundle.js"
}

// Synchronously delete the output script file(s)
gulp.task("clean-scripts", function (cb) {
    del(["lib","dist"], cb);
});

//Create a jquery bundled file
gulp.task("jquery-bundle", ["clean-scripts", "bower-restore"], function () {
    return gulp.src(config.jquerysrc)
     .pipe(concat("jquery-bundle.js"))
     .pipe(gulp.dest("lib"));
});

//Create a angular bundled file
gulp.task("ng-bundle", ["clean-scripts", "bower-restore"], function () {
    return gulp.src(config.ngsrc)
     .pipe(concat("ng-bundle.js"))
     .pipe(gulp.dest("lib"));
});

//Create a bootstrap bundled file
gulp.task("bootstrap-bundle", ["clean-scripts", "bower-restore"], function     () {
    return gulp.src(config.bootstrapsrc)
     .pipe(concat("bootstrap-bundle.js"))
     .pipe(gulp.dest("lib"));
});


// Combine and the vendor files from bower into bundles (output to the Scripts folder)
gulp.task("bundle-scripts", ["jquery-bundle", "ng-bundle", "bootstrap-bundle"], function () {

});

//Restore all bower packages
gulp.task("bower-restore", function () {
    return bower();
});

//build lib scripts
gulp.task("compile-lib", ["bundle-scripts"], function () {
    return gulp.src("lib/*.js")
        .pipe(sourcemaps.init())
        .pipe(concat("compiled-bundle.js"))
        .pipe(gulp.dest("dist"))
        .pipe(rename("compiled-bundle.min.js"))
        .pipe(uglify())
        .pipe(sourcemaps.write("./"))
        .pipe(gulp.dest("dist"));
});

How do you see the entire command history in interactive Python?

With python 3 interpreter the history is written to
~/.python_history

Laravel Eloquent - Get one Row

laravel 5.8

If you don't even need an entire row, you may extract a single value from a record using the value() method. This method will return the value of the column directly:

$first_name = DB::table('users')->where('email' ,'me@mail,com')->value('first_name');

check docs

How do browser cookie domains work?

There are rules that determine whether a browser will accept the Set-header response header (server-side cookie writing), a slightly different rules/interpretations for cookie set using Javascript (I haven't tested VBScript).

Then there are rules that determine whether the browser will send a cookie along with the page request.

There are differences between the major browser engines how domain matches are handled, and how parameters in path values are interpreted. You can find some empirical evidence in the article How Different Browsers Handle Cookies Differently

How add "or" in switch statements?

Case-statements automatically fall through if you don't specify otherwise (by writing break). Therefor you can write

switch(myvar)
{
   case 2:
   case 5:
   {
      //your code
   break;
   }

// etc... }

What is the difference between a "function" and a "procedure"?

In most contexts: a function returns a value, while a procedure doesn't. Both are pieces of code grouped together to do the same thing.

In functional programming context (where all functions return values), a function is an abstract object:

f(x)=(1+x)
g(x)=.5*(2+x/2)

Here, f is the same function as g, but is a different procedure.

Understanding `scale` in R

I thought I would contribute by providing a concrete example of the practical use of the scale function. Say you have 3 test scores (Math, Science, and English) that you want to compare. Maybe you may even want to generate a composite score based on each of the 3 tests for each observation. Your data could look as as thus:

student_id <- seq(1,10)
math <- c(502,600,412,358,495,512,410,625,573,522)
science <- c(95,99,80,82,75,85,80,95,89,86)
english <- c(25,22,18,15,20,28,15,30,27,18)
df <- data.frame(student_id,math,science,english)

Obviously it would not make sense to compare the means of these 3 scores as the scale of the scores are vastly different. By scaling them however, you have more comparable scoring units:

z <- scale(df[,2:4],center=TRUE,scale=TRUE)

You could then use these scaled results to create a composite score. For instance, average the values and assign a grade based on the percentiles of this average. Hope this helped!

Note: I borrowed this example from the book "R In Action". It's a great book! Would definitely recommend.

How to build an APK file in Eclipse?

When you run your application, your phone should be detected and you should be given the option to run on your phone instead of on the emulator.

More instructions on getting your phone recognized: http://developer.android.com/guide/developing/device.html

When you want to export a signed version of the APK file (for uploading to the market or putting on a website), right-click on the project in Eclipse, choose export, and then choose "Export Android Application".

More details: http://developer.android.com/guide/publishing/app-signing.html#ExportWizard

How to install popper.js with Bootstrap 4?

Try doing this:

npm install bootstrap jquery popper.js --save

See this page for more information: how-to-include-bootstrap-in-your-project-with-webpack

Simplest PHP example for retrieving user_timeline with Twitter API version 1.1

This question helped me a lot but didn't get me all the way in understanding what needs to happen. This blog post did an amazing job of walking me through it.

Here are the important bits all in one place:

  • As pointed out above, you MUST sign your 1.1 API requests. If you are doing something like getting public statuses, you'll want an application key rather than a user key. The full link to the page you want is: https://dev.twitter.com/apps
  • You must hash ALL the parameters, both the oauth ones AND the get parameters (or POST parameters) together.
  • You must SORT the parameters before reducing them to the url encoded form that gets hashed.
  • You must encode some things multiple times - for example, you create a query string from the parameters' url-encoded values, and then you url encode THAT and concatenate with the method type and the url.

I sympathize with all the headaches, so here's some code to wrap it all up:

$token = 'YOUR TOKEN';
$token_secret = 'TOKEN SECRET';
$consumer_key = 'YOUR KEY';
$consumer_secret = 'KEY SECRET';

$host = 'api.twitter.com';
$method = 'GET';
$path = '/1.1/statuses/user_timeline.json'; // api call path

$query = array( // query parameters
    'screen_name' => 'twitterapi',
    'count' => '2'
);

$oauth = array(
    'oauth_consumer_key' => $consumer_key,
    'oauth_token' => $token,
    'oauth_nonce' => (string)mt_rand(), // a stronger nonce is recommended
    'oauth_timestamp' => time(),
    'oauth_signature_method' => 'HMAC-SHA1',
    'oauth_version' => '1.0'
);

$oauth = array_map("rawurlencode", $oauth); // must be encoded before sorting
$query = array_map("rawurlencode", $query);

$arr = array_merge($oauth, $query); // combine the values THEN sort

asort($arr); // secondary sort (value)
ksort($arr); // primary sort (key)

// http_build_query automatically encodes, but our parameters
// are already encoded, and must be by this point, so we undo
// the encoding step
$querystring = urldecode(http_build_query($arr, '', '&'));

$url = "https://$host$path";

// mash everything together for the text to hash
$base_string = $method."&".rawurlencode($url)."&".rawurlencode($querystring);

// same with the key
$key = rawurlencode($consumer_secret)."&".rawurlencode($token_secret);

// generate the hash
$signature = rawurlencode(base64_encode(hash_hmac('sha1', $base_string, $key, true)));

// this time we're using a normal GET query, and we're only encoding the query params
// (without the oauth params)
$url .= "?".http_build_query($query);

$oauth['oauth_signature'] = $signature; // don't want to abandon all that work!
ksort($oauth); // probably not necessary, but twitter's demo does it

// also not necessary, but twitter's demo does this too
function add_quotes($str) { return '"'.$str.'"'; }
$oauth = array_map("add_quotes", $oauth);

// this is the full value of the Authorization line
$auth = "OAuth " . urldecode(http_build_query($oauth, '', ', '));

// if you're doing post, you need to skip the GET building above
// and instead supply query parameters to CURLOPT_POSTFIELDS
$options = array( CURLOPT_HTTPHEADER => array("Authorization: $auth"),
                  //CURLOPT_POSTFIELDS => $postfields,
                  CURLOPT_HEADER => false,
                  CURLOPT_URL => $url,
                  CURLOPT_RETURNTRANSFER => true,
                  CURLOPT_SSL_VERIFYPEER => false);

// do our business
$feed = curl_init();
curl_setopt_array($feed, $options);
$json = curl_exec($feed);
curl_close($feed);

$twitter_data = json_decode($json);

surface plots in matplotlib

This is not a general solution but might help many of those who just typed "matplotlib surface plot" in Google and landed here.

Suppose you have data = [(x1,y1,z1),(x2,y2,z2),.....,(xn,yn,zn)], then you can get three 1-d lists using x, y, z = zip(*data). Now you can of course create 3d scatterplot using three 1-d lists.

But, why can't in general this data be used to create surface plot? To understand that consider an empty 3-d plot :

Now, suppose for each possible value of (x, y) on a "discrete" regular grid, you have a z value, then there's no issue & you can in fact get a surface plot:

import numpy as np
from matplotlib import pyplot as plt
from mpl_toolkits.mplot3d import Axes3D
from matplotlib import cm

x = np.linspace(0, 10, 6)  # [0, 2,..,10] : 6 distinct values
y = np.linspace(0, 20, 5)  # [0, 5,..,20] : 5 distinct values
z = np.linspace(0, 100, 30)  # 6 * 5 = 30 values, 1 for each possible combination of (x,y)

X, Y = np.meshgrid(x, y)
Z = np.reshape(z, X.shape)  # Z.shape must be equal to X.shape = Y.shape

fig = plt.figure()
ax = fig.add_subplot(111, projection='3d')

ax.plot_surface(X, Y, Z)

ax.set_xlabel('X Label')
ax.set_ylabel('Y Label')
ax.set_zlabel('Z Label')
plt.show()

What's happens when you haven't got z for all possible combinations of (x, y)? Then at the point (at intersection of two black lines on x-y plane on blank plot above), we don't know what is the value of z. It could be anything, we don't know how 'high' or 'low' our surface should be at that point (although it can be approximated using other functions, surface_plot requires that you supply it arguments where X.shape = Y.shape = Z.shape).

Java GUI frameworks. What to choose? Swing, SWT, AWT, SwingX, JGoodies, JavaFX, Apache Pivot?

I would like to suggest another framework: Apache Pivot http://pivot.apache.org/.

I tried it briefly and was impressed by what it can offer as an RIA (Rich Internet Application) framework ala Flash.

It renders UI using Java2D, thus minimizing the impact of (IMO, bloated) legacies of Swing and AWT.

How to use curl to get a GET request exactly same as using Chrome?

Open Chrome Developer Tools, go to Network tab, make your request (you may need to check "Preserve Log" if the page refreshes). Find the request on the left, right-click, "Copy as cURL".

Convert String to SecureString

If you would like to compress the conversion of a string to a SecureString into a LINQ statement you can express it as follows:

var plain  = "The quick brown fox jumps over the lazy dog";
var secure = plain
             .ToCharArray()
             .Aggregate( new SecureString()
                       , (s, c) => { s.AppendChar(c); return s; }
                       , (s)    => { s.MakeReadOnly(); return s; }
                       );

However, keep in mind that using LINQ does not improve the security of this solution. It suffers from the same flaw as any conversion from string to SecureString. As long as the original string remains in memory the data is vulnerable.

That being said, what the above statement can offer is keeping together the creation of the SecureString, its initialization with data and finally locking it from modification.

How do I get the color from a hexadecimal color code using .NET?

I used ColorDialog in my project. ColorDialog sometimess return "Red","Fhushia" and sometimes return "fff000". I solved this problem like this maybe help someone.

        SolidBrush guideLineColor;
        if (inputColor.Any(c => char.IsDigit(c)))
        {
            string colorcode = inputColor;
            int argbInputColor = Int32.Parse(colorcode.Replace("#", ""), NumberStyles.HexNumber);
             guideLineColor = new SolidBrush(Color.FromArgb(argbInputColor));

        }
        else
        {
            Color col = Color.FromName(inputColor);
             guideLineColor = new SolidBrush(col);
        }

InputColor is the return value from ColorDialog.

Thanks everyone for answer this question.It's big help to me.

How to specify names of columns for x and y when joining in dplyr?

This feature has been added in dplyr v0.3. You can now pass a named character vector to the by argument in left_join (and other joining functions) to specify which columns to join on in each data frame. With the example given in the original question, the code would be:

left_join(test_data, kantrowitz, by = c("first_name" = "name"))

Remove first Item of the array (like popping from stack)

The easiest way is using shift(). If you have an array, the shift function shifts everything to the left.

var arr = [1, 2, 3, 4]; 
var theRemovedElement = arr.shift(); // theRemovedElement == 1
console.log(arr); // [2, 3, 4]

How can I read comma separated values from a text file in Java?

Use OpenCSV for reliability. Split should never be used for these kind of things. Here's a snippet from a program of my own, it's pretty straightforward. I check if a delimiter character was specified and use this one if it is, if not I use the default in OpenCSV (a comma). Then i read the header and fields

CSVReader reader = null;
try {
    if (delimiter > 0) {
        reader = new CSVReader(new FileReader(this.csvFile), this.delimiter);
    }
    else {
        reader = new CSVReader(new FileReader(this.csvFile));
    }

    // these should be the header fields
    header = reader.readNext();
    while ((fields = reader.readNext()) != null) {
        // more code
    }
catch (IOException e) {
    System.err.println(e.getMessage());
}

How do I bottom-align grid elements in bootstrap fluid layout

Well, I didn't like any of those answers, my solution of the same problem was to add this:<div>&nbsp;</div>. So in your scheme it would look like this (more or less), no style changes were necessary in my case:

-row-fluid-------------------------------------
+-span6----------+ +----span6----------+
|                | | +---div---+       |
| content        | | | & nbsp; |       |
| that           | | +---------+       |
| is tall        | | +-----div--------+|   
|                | | |short content   ||
|                | | +----------------+|
+----------------+ +-------------------+
-----------------------------------------------

Insert results of a stored procedure into a temporary table

It's a simple 2 step process: - create a temporary table - Insert into the temporary table.

Code to perform the same:

CREATE TABLE #tempTable (Column1 int, Column2 varchar(max));
INSERT INTO #tempTable 
EXEC [app].[Sproc_name]
@param1 = 1,
@param2 =2;

What's the use of "enum" in Java?

1) enum is a keyword in Object oriented method.

2) It is used to write the code in a Single line, That's it not more than that.

     public class NAME
     {
        public static final String THUNNE = "";
        public static final String SHAATA = ""; 
        public static final String THULLU = ""; 
     }

-------This can be replaced by--------

  enum NAME{THUNNE, SHAATA, THULLU}

3) Most of the developers do not use enum keyword, it is just a alternative method..

calculating number of days between 2 columns of dates in data frame

Without your seeing your data (you can use the output of dput(head(survey)) to show us) this is a shot in the dark:

survey <- data.frame(date=c("2012/07/26","2012/07/25"),tx_start=c("2012/01/01","2012/01/01"))

survey$date_diff <- as.Date(as.character(survey$date), format="%Y/%m/%d")-
                  as.Date(as.character(survey$tx_start), format="%Y/%m/%d")
survey
       date   tx_start date_diff
1 2012/07/26 2012/01/01  207 days
2 2012/07/25 2012/01/01  206 days

Set Memory Limit in htaccess

In your .htaccess you can add:

PHP 5.x

<IfModule mod_php5.c>
    php_value memory_limit 64M
</IfModule>

PHP 7.x

<IfModule mod_php7.c>
    php_value memory_limit 64M
</IfModule>

If page breaks again, then you are using PHP as mod_php in apache, but error is due to something else.

If page does not break, then you are using PHP as CGI module and therefore cannot use php values - in the link I've provided might be solution but I'm not sure you will be able to apply it.

Read more on http://support.tigertech.net/php-value

How to install pywin32 module in windows 7

are you just trying to install it, or are you looking to build from source?

If you just need to install, the easiest way is to use the MSI installers provided here:

http://sourceforge.net/projects/pywin32/files/pywin32/ (for updated versions)

make sure you get the correct version (matches Python version, 32bit/64bit, etc)

This Handler class should be static or leaks might occur: IncomingHandler

Here is a generic example of using a weak reference and static handler class to resolve the problem (as recommended in the Lint documentation):

public class MyClass{

  //static inner class doesn't hold an implicit reference to the outer class
  private static class MyHandler extends Handler {
    //Using a weak reference means you won't prevent garbage collection
    private final WeakReference<MyClass> myClassWeakReference; 

    public MyHandler(MyClass myClassInstance) {
      myClassWeakReference = new WeakReference<MyClass>(myClassInstance);
    }

    @Override
    public void handleMessage(Message msg) {
      MyClass myClass = myClassWeakReference.get();
      if (myClass != null) {
        ...do work here...
      }
    }
  }

  /**
   * An example getter to provide it to some external class
   * or just use 'new MyHandler(this)' if you are using it internally.
   * If you only use it internally you might even want it as final member:
   * private final MyHandler mHandler = new MyHandler(this);
   */
  public Handler getHandler() {
    return new MyHandler(this);
  }
}

proper name for python * operator?

I say "star-args" and Python people seem to know what i mean.

** is trickier - I think just "qargs" since it is usually used as **kw or **kwargs

Can You Get A Users Local LAN IP Address Via JavaScript?

Chrome 76+

Last year I used Linblow's answer (2018-Oct-19) to successfully discover my local IP via javascript. However, recent Chrome updates (76?) have wonked this method so that it now returns an obfuscated IP, such as: 1f4712db-ea17-4bcf-a596-105139dfd8bf.local

If you have full control over your browser, you can undo this behavior by turning it off in Chrome Flags, by typing this into your address bar:

chrome://flags

and DISABLING the flag Anonymize local IPs exposed by WebRTC

In my case, I require the IP for a TamperMonkey script to determine my present location and do different things based on my location. I also have full control over my own browser settings (no Corporate Policies, etc). So for me, changing the chrome://flags setting does the trick.

Sources:

https://groups.google.com/forum/#!topic/discuss-webrtc/6stQXi72BEU

https://codelabs.developers.google.com/codelabs/webrtc-web/index.html

Image height and width not working?

http://www.markrafferty.com/wp-content/w3tc/min/7415c412.e68ae1.css

Line 11:

.postItem img {
    height: auto;
    width: 450px;
}

You can either edit your CSS, or you can listen to Mageek and use INLINE STYLING to override the CSS styling that's happening:

<img src="theSource" style="width:30px;" />

Avoid setting both width and height, as the image itself might not be scaled proportionally. But you can set the dimensions to whatever you want, as per Mageek's example.

SQL Server : converting varchar to INT

This is more for someone Searching for a result, than the original post-er. This worked for me...

declare @value varchar(max) = 'sad';
select sum(cast(iif(isnumeric(@value) = 1, @value, 0) as bigint));

returns 0

declare @value varchar(max) = '3';
select sum(cast(iif(isnumeric(@value) = 1, @value, 0) as bigint));

returns 3

Does Java have an exponential operator?

To do this with user input:

public static void getPow(){
    Scanner sc = new Scanner(System.in);
    System.out.println("Enter first integer: ");    // 3
    int first = sc.nextInt();
    System.out.println("Enter second integer: ");    // 2
    int second = sc.nextInt();
    System.out.println(first + " to the power of " + second + " is " + 
        (int) Math.pow(first, second));    // outputs 9

Does calling clone() on an array also clone its contents?

1D array of primitives does copy elements when it is cloned. This tempts us to clone 2D array(Array of Arrays).

Remember that 2D array clone doesn't work due to shallow copy implementation of clone().

public static void main(String[] args) {
    int row1[] = {0,1,2,3};
    int row2[] =  row1.clone();
    row2[0] = 10;
    System.out.println(row1[0] == row2[0]); // prints false

    int table1[][]={{0,1,2,3},{11,12,13,14}};
    int table2[][] = table1.clone();
    table2[0][0] = 100;
    System.out.println(table1[0][0] == table2[0][0]); //prints true
}

How to use Git and Dropbox together?

Without using third-party integration tools, I could enhance the condition a bit and use DropBox and other similar cloud disk services such as SpiderOak with Git.

The goal is to avoid the synchronization in the middle of these files modifications, as it can upload a partial state and will then download it back, completely corrupting your git state.

To avoid this issue, I did:

  1. Bundle my git index in one file using git bundle create my_repo.git --all.
  2. Set a delay for the file monitoring, eg 5 minutes, instead of instantaneous. This reduces the chances DropBox synchronizes a partial state in the middle of a change. It also helps greatly when modifying files on the cloud disk on-the-fly (such as with instantaneous saving note-taking apps).

It's not perfect as there is no guarantee it won't mess up the git state again, but it helps and for the moment I did not get any issue.

'AND' vs '&&' as operator

Here's a little counter example:

$a = true;
$b = true;
$c = $a & $b;
var_dump(true === $c);

output:

bool(false)

I'd say this kind of typo is far more likely to cause insidious problems (in much the same way as = vs ==) and is far less likely to be noticed than adn/ro typos which will flag as syntax errors. I also find and/or is much easier to read. FWIW, most PHP frameworks that express a preference (most don't) specify and/or. I've also never run into a real, non-contrived case where it would have mattered.

typescript: error TS2693: 'Promise' only refers to a type, but is being used as a value here

I had the same problem and this saved me from the problem in second:

write in console this:

npm i --save bluebird
npm i --save-dev @types/bluebird @types/[email protected]

in the file where the problem is copy paste this:

import * as Promise from 'bluebird';

Create a root password for PHPMyAdmin

I just faced the mysql user password problem - ERROR 1045: Access denied for user: 'root@localhost' (Using password: NO) - when I tried to do a do-release-upgrade on my operational system. So I corrected it in 2 steps.

Firstly, as I did not had access on phpmyadmin, so I followed the "Recover MySQL root password" step on the tutorial mensioned by ThoKra: https://www.howtoforge.com/setting-changing-resetting-mysql-root-passwords

Secondly, with one of the users that I know the password, I made some password changes to the others users through phpmyadmin itself according to SonDang's information.

Redirecting a page using Javascript, like PHP's Header->Location

You cannot mix JS and PHP that way, PHP is rendered before the page is sent to the browser (i.e. before the JS is run)

You can use window.location to change your current page.

$('.entry a:first').click(function() {
    window.location = "http://google.ca";
});

Best way to update an element in a generic List

If the list is sorted (as happens to be in the example) a binary search on index certainly works.

    public static Dog Find(List<Dog> AllDogs, string Id)
    {
        int p = 0;
        int n = AllDogs.Count;
        while (true)
        {
            int m = (n + p) / 2;
            Dog d = AllDogs[m];
            int r = string.Compare(Id, d.Id);
            if (r == 0)
                return d;
            if (m == p)
                return null;
            if (r < 0)
                n = m;
            if (r > 0)
                p = m;
        }
    }

Not sure what the LINQ version of this would be.

Catch an exception thrown by an async void method

Your code doesn't do what you might think it does. Async methods return immediately after the method begins waiting for the async result. It's insightful to use tracing in order to investigate how the code is actually behaving.

The code below does the following:

  • Create 4 tasks
  • Each task will asynchronously increment a number and return the incremented number
  • When the async result has arrived it is traced.

 

static TypeHashes _type = new TypeHashes(typeof(Program));        
private void Run()
{
    TracerConfig.Reset("debugoutput");

    using (Tracer t = new Tracer(_type, "Run"))
    {
        for (int i = 0; i < 4; i++)
        {
            DoSomeThingAsync(i);
        }
    }
    Application.Run();  // Start window message pump to prevent termination
}


private async void DoSomeThingAsync(int i)
{
    using (Tracer t = new Tracer(_type, "DoSomeThingAsync"))
    {
        t.Info("Hi in DoSomething {0}",i);
        try
        {
            int result = await Calculate(i);
            t.Info("Got async result: {0}", result);
        }
        catch (ArgumentException ex)
        {
            t.Error("Got argument exception: {0}", ex);
        }
    }
}

Task<int> Calculate(int i)
{
    var t = new Task<int>(() =>
    {
        using (Tracer t2 = new Tracer(_type, "Calculate"))
        {
            if( i % 2 == 0 )
                throw new ArgumentException(String.Format("Even argument {0}", i));
            return i++;
        }
    });
    t.Start();
    return t;
}

When you observe the traces

22:25:12.649  02172/02820 {          AsyncTest.Program.Run 
22:25:12.656  02172/02820 {          AsyncTest.Program.DoSomeThingAsync     
22:25:12.657  02172/02820 Information AsyncTest.Program.DoSomeThingAsync Hi in DoSomething 0    
22:25:12.658  02172/05220 {          AsyncTest.Program.Calculate    
22:25:12.659  02172/02820 {          AsyncTest.Program.DoSomeThingAsync     
22:25:12.659  02172/02820 Information AsyncTest.Program.DoSomeThingAsync Hi in DoSomething 1    
22:25:12.660  02172/02756 {          AsyncTest.Program.Calculate    
22:25:12.662  02172/02820 {          AsyncTest.Program.DoSomeThingAsync     
22:25:12.662  02172/02820 Information AsyncTest.Program.DoSomeThingAsync Hi in DoSomething 2    
22:25:12.662  02172/02820 {          AsyncTest.Program.DoSomeThingAsync     
22:25:12.662  02172/02820 Information AsyncTest.Program.DoSomeThingAsync Hi in DoSomething 3    
22:25:12.664  02172/02756          } AsyncTest.Program.Calculate Duration 4ms   
22:25:12.666  02172/02820          } AsyncTest.Program.Run Duration 17ms  ---- Run has completed. The async methods are now scheduled on different threads. 
22:25:12.667  02172/02756 Information AsyncTest.Program.DoSomeThingAsync Got async result: 1    
22:25:12.667  02172/02756          } AsyncTest.Program.DoSomeThingAsync Duration 8ms    
22:25:12.667  02172/02756 {          AsyncTest.Program.Calculate    
22:25:12.665  02172/05220 Exception   AsyncTest.Program.Calculate Exception thrown: System.ArgumentException: Even argument 0   
   at AsyncTest.Program.c__DisplayClassf.Calculateb__e() in C:\Source\AsyncTest\AsyncTest\Program.cs:line 124   
   at System.Threading.Tasks.Task`1.InvokeFuture(Object futureAsObj)    
   at System.Threading.Tasks.Task.InnerInvoke()     
   at System.Threading.Tasks.Task.Execute()     
22:25:12.668  02172/02756 Exception   AsyncTest.Program.Calculate Exception thrown: System.ArgumentException: Even argument 2   
   at AsyncTest.Program.c__DisplayClassf.Calculateb__e() in C:\Source\AsyncTest\AsyncTest\Program.cs:line 124   
   at System.Threading.Tasks.Task`1.InvokeFuture(Object futureAsObj)    
   at System.Threading.Tasks.Task.InnerInvoke()     
   at System.Threading.Tasks.Task.Execute()     
22:25:12.724  02172/05220          } AsyncTest.Program.Calculate Duration 66ms      
22:25:12.724  02172/02756          } AsyncTest.Program.Calculate Duration 57ms      
22:25:12.725  02172/05220 Error       AsyncTest.Program.DoSomeThingAsync Got argument exception: System.ArgumentException: Even argument 0  

Server stack trace:     
   at AsyncTest.Program.c__DisplayClassf.Calculateb__e() in C:\Source\AsyncTest\AsyncTest\Program.cs:line 124   
   at System.Threading.Tasks.Task`1.InvokeFuture(Object futureAsObj)    
   at System.Threading.Tasks.Task.InnerInvoke()     
   at System.Threading.Tasks.Task.Execute()     

Exception rethrown at [0]:      
   at System.Runtime.CompilerServices.TaskAwaiter.EndAwait()    
   at System.Runtime.CompilerServices.TaskAwaiter`1.EndAwait()  
   at AsyncTest.Program.DoSomeThingAsyncd__8.MoveNext() in C:\Source\AsyncTest\AsyncTest\Program.cs:line 106    
22:25:12.725  02172/02756 Error       AsyncTest.Program.DoSomeThingAsync Got argument exception: System.ArgumentException: Even argument 2  

Server stack trace:     
   at AsyncTest.Program.c__DisplayClassf.Calculateb__e() in C:\Source\AsyncTest\AsyncTest\Program.cs:line 124   
   at System.Threading.Tasks.Task`1.InvokeFuture(Object futureAsObj)    
   at System.Threading.Tasks.Task.InnerInvoke()     
   at System.Threading.Tasks.Task.Execute()     

Exception rethrown at [0]:      
   at System.Runtime.CompilerServices.TaskAwaiter.EndAwait()    
   at System.Runtime.CompilerServices.TaskAwaiter`1.EndAwait()  
   at AsyncTest.Program.DoSomeThingAsyncd__8.MoveNext() in C:\Source\AsyncTest\AsyncTest\Program.cs:line 0      
22:25:12.726  02172/05220          } AsyncTest.Program.DoSomeThingAsync Duration 70ms   
22:25:12.726  02172/02756          } AsyncTest.Program.DoSomeThingAsync Duration 64ms   
22:25:12.726  02172/05220 {          AsyncTest.Program.Calculate    
22:25:12.726  02172/05220          } AsyncTest.Program.Calculate Duration 0ms   
22:25:12.726  02172/05220 Information AsyncTest.Program.DoSomeThingAsync Got async result: 3    
22:25:12.726  02172/05220          } AsyncTest.Program.DoSomeThingAsync Duration 64ms   

You will notice that the Run method completes on thread 2820 while only one child thread has finished (2756). If you put a try/catch around your await method you can "catch" the exception in the usual way although your code is executed on another thread when the calculation task has finished and your contiuation is executed.

The calculation method traces the thrown exception automatically because I did use the ApiChange.Api.dll from the ApiChange tool. Tracing and Reflector helps a lot to understand what is going on. To get rid of threading you can create your own versions of GetAwaiter BeginAwait and EndAwait and wrap not a task but e.g. a Lazy and trace inside your own extension methods. Then you will get much better understanding what the compiler and what the TPL does.

Now you see that there is no way to get in a try/catch your exception back since there is no stack frame left for any exception to propagate from. Your code might be doing something totally different after you did initiate the async operations. It might call Thread.Sleep or even terminate. As long as there is one foreground thread left your application will happily continue to execute asynchronous tasks.


You can handle the exception inside the async method after your asynchronous operation did finish and call back into the UI thread. The recommended way to do this is with TaskScheduler.FromSynchronizationContext. That does only work if you have an UI thread and it is not very busy with other things.

How can I wait In Node.js (JavaScript)? l need to pause for a period of time

For more info on

yield sleep(2000); 

you should check Redux-Saga. But it is specific to your choice of Redux as your model framework (although strictly not necessary).

How to specify the current directory as path in VBA?

If the path you want is the one to the workbook running the macro, and that workbook has been saved, then

ThisWorkbook.Path

is what you would use.

file_get_contents() Breaks Up UTF-8 Characters

Try this too

 $url = 'http://www.domain.com/';
    $html = file_get_contents($url);

    //Change encoding to UTF-8 from ISO-8859-1
    $html = iconv('UTF-8', 'ISO-8859-1//TRANSLIT', $html);

Case Statement Equivalent in R

Imho, most straightforward and universal code:

dft=data.frame(x = sample(letters[1:8], 20, replace=TRUE))
dft=within(dft,{
    y=NA
    y[x %in% c('a','b','c')]='abc'
    y[x %in% c('d','e','f')]='def'
    y[x %in% 'g']='g'
    y[x %in% 'h']='h'
})

How to get current class name including package name in Java?

Use this.getClass().getCanonicalName() to get the full class name.

Note that a package / class name ("a.b.C") is different from the path of the .class files (a/b/C.class), and that using the package name / class name to derive a path is typically bad practice. Sets of class files / packages can be in multiple different class paths, which can be directories or jar files.

Google Maps API v3: Can I setZoom after fitBounds?

If I'm not mistaken, I'm assuming you want all your points to be visible on the map with the highest possible zoom level. I accomplished this by initializing the zoom level of the map to 16(not sure if it's the highest possible zoom level on V3).

var map = new google.maps.Map(document.getElementById('map_canvas'), {
  zoom: 16,
  center: marker_point,
  mapTypeId: google.maps.MapTypeId.ROADMAP
});

Then after that I did the bounds stuff:

var bounds = new google.maps.LatLngBounds();

// You can have a loop here of all you marker points
// Begin loop
bounds.extend(marker_point);
// End loop

map.fitBounds(bounds);

Result: Success!

Iterating on a file doesn't work the second time

Of course. That is normal and sane behaviour. Instead of closing and re-opening, you could rewind the file.

How to hide/show more text within a certain length (like youtube)

Show More, Show Less (Only when needed) No JQuery

I needed this functionality for an RSS feed on our company's website. This is what I came up with:

_x000D_
_x000D_
// Create Variables_x000D_
var allOSB = [];_x000D_
var mxh = '';_x000D_
_x000D_
window.onload = function() {_x000D_
  // Set Variables_x000D_
  allOSB = document.getElementsByClassName("only-so-big");_x000D_
  _x000D_
  if (allOSB.length > 0) {_x000D_
    mxh = window.getComputedStyle(allOSB[0]).getPropertyValue('max-height');_x000D_
    mxh = parseInt(mxh.replace('px', ''));_x000D_
    _x000D_
    // Add read-more button to each OSB section_x000D_
    for (var i = 0; i < allOSB.length; i++) {_x000D_
      var el = document.createElement("button");_x000D_
      el.innerHTML = "Read More";_x000D_
      el.setAttribute("type", "button");_x000D_
      el.setAttribute("class", "read-more hid");_x000D_
      _x000D_
      insertAfter(allOSB[i], el);_x000D_
    }_x000D_
  }_x000D_
_x000D_
  // Add click function to buttons_x000D_
  var readMoreButtons = document.getElementsByClassName("read-more");_x000D_
  for (var i = 0; i < readMoreButtons.length; i++) {_x000D_
    readMoreButtons[i].addEventListener("click", function() { _x000D_
      revealThis(this);_x000D_
    }, false);_x000D_
  }_x000D_
  _x000D_
  // Update buttons so only the needed ones show_x000D_
  updateReadMore();_x000D_
}_x000D_
// Update on resize_x000D_
window.onresize = function() {_x000D_
  updateReadMore();_x000D_
}_x000D_
_x000D_
// show only the necessary read-more buttons_x000D_
function updateReadMore() {_x000D_
  if (allOSB.length > 0) {_x000D_
    for (var i = 0; i < allOSB.length; i++) {_x000D_
      if (allOSB[i].scrollHeight > mxh) {_x000D_
        if (allOSB[i].hasAttribute("style")) {_x000D_
          updateHeight(allOSB[i]);_x000D_
        }_x000D_
        allOSB[i].nextElementSibling.className = "read-more";_x000D_
      } else {_x000D_
        allOSB[i].nextElementSibling.className = "read-more hid";_x000D_
      }_x000D_
    }_x000D_
  }_x000D_
}_x000D_
_x000D_
function revealThis(current) {_x000D_
  var el = current.previousElementSibling;_x000D_
  if (el.hasAttribute("style")) {_x000D_
    current.innerHTML = "Read More";_x000D_
    el.removeAttribute("style");_x000D_
  } else {_x000D_
    updateHeight(el);_x000D_
    current.innerHTML = "Show Less";_x000D_
  }_x000D_
}_x000D_
_x000D_
function updateHeight(el) {_x000D_
  el.style.maxHeight = el.scrollHeight + "px";_x000D_
}_x000D_
_x000D_
// thanks to karim79 for this function_x000D_
// http://stackoverflow.com/a/4793630/5667951_x000D_
function insertAfter(referenceNode, newNode) {_x000D_
    referenceNode.parentNode.insertBefore(newNode, referenceNode.nextSibling);_x000D_
}
_x000D_
@import url('https://fonts.googleapis.com/css?family=Open+Sans:600');_x000D_
_x000D_
*,_x000D_
*:before,_x000D_
*:after {_x000D_
  box-sizing: border-box;_x000D_
}_x000D_
html,_x000D_
body {_x000D_
  height: 100%;_x000D_
  margin: 0;_x000D_
  padding: 0;_x000D_
  width: 100%;_x000D_
}_x000D_
body {_x000D_
  font-family: 'Open Sans', sans-serif;_x000D_
}_x000D_
h1 {_x000D_
  text-align: center;_x000D_
}_x000D_
.main-container {_x000D_
  margin: 30px auto;_x000D_
  max-width: 1000px;_x000D_
  padding: 20px;_x000D_
}_x000D_
.only-so-big p {_x000D_
  padding: 0;_x000D_
  margin: 0;_x000D_
}_x000D_
p {_x000D_
  font-size: 15px;_x000D_
  line-height: 20px;_x000D_
}_x000D_
hr {_x000D_
  background: #ccc;_x000D_
  display: block;_x000D_
  height: 1px;_x000D_
  width: 100%;_x000D_
}_x000D_
_x000D_
_x000D_
/*_x000D_
  NECESSARY SECTION_x000D_
*/_x000D_
.only-so-big {_x000D_
  background: rgba(178, 252, 255, .3);_x000D_
  height: 100%;_x000D_
  max-height: 100px;_x000D_
  overflow: hidden;_x000D_
  -webkit-transition: max-height .75s;_x000D_
  transition: max-height .75s;_x000D_
}_x000D_
_x000D_
.read-more {_x000D_
  background: none;_x000D_
  border: none;_x000D_
  color: #1199f9;_x000D_
  cursor: pointer;_x000D_
  font-size: 1em;_x000D_
  outline: none; _x000D_
}_x000D_
.read-more:hover {_x000D_
  text-decoration: underline;_x000D_
}_x000D_
.read-more:focus {_x000D_
  outline: none;_x000D_
}_x000D_
.read-more::-moz-focus-inner {_x000D_
  border: 0;_x000D_
}_x000D_
.hid {_x000D_
  display: none;_x000D_
}
_x000D_
<div class="main-container">_x000D_
      <h1>Controlling Different Content Size</h1>_x000D_
      <p>We're working with an RSS feed and have had issues with some being much larger than others (content wise). I only want to show the "Read More" button if it's needed.</p>_x000D_
      <div class="only-so-big">_x000D_
        <p>This is just a short guy. No need for the read more button.</p>_x000D_
      </div>_x000D_
      <hr>_x000D_
      <div class="only-so-big">_x000D_
        <p>This one has way too much content to show. Best be saving it for those who want to read everything in here.</p>_x000D_
        <p>Sed ut perspiciatis unde omnis iste natus error sit voluptatem accusantium doloremque laudantium, totam rem aperiam, eaque ipsa quae ab illo inventore veritatis et quasi architecto beatae vitae dicta sunt explicabo. Nemo enim ipsam voluptatem quia_x000D_
          voluptas sit aspernatur aut odit aut fugit, sed quia consequuntur magni dolores eos qui ratione voluptatem sequi nesciunt. Neque porro quisquam est, qui dolorem ipsum quia dolor sit amet, consectetur, adipisci velit, sed quia non numquam eius modi THE END!</p>_x000D_
      </div>_x000D_
      <hr>_x000D_
      <div class="only-so-big">_x000D_
        <p>Another small section with not a lot of content</p>_x000D_
      </div>_x000D_
      <hr>_x000D_
      <div class="only-so-big">_x000D_
        <p>Make Window smaller to show "Read More" button.<br> totam rem aperiam, eaque ipsa quae ab illo inventore veritatis et quasi architecto beatae vitae dicta sunt explicabo. Nemo enim ipsam voluptatem quia voluptas sit aspernatur aut odit aut fugit, sed?</p>_x000D_
      </div>_x000D_
    </div>
_x000D_
_x000D_
_x000D_

splitting a number into the integer and decimal parts

If you don't mind using NumPy, then:

In [319]: real = np.array([1234.5678])

In [327]: integ, deci = int(np.floor(real)), np.asscalar(real % 1)

In [328]: integ, deci
Out[328]: (1234, 0.5678000000000338)

How can I verify if an AD account is locked?

Here's another one:

PS> Search-ADAccount -Locked | Select Name, LockedOut, LastLogonDate

Name                                       LockedOut LastLogonDate
----                                       --------- -------------
Yxxxxxxx                                        True 14/11/2014 10:19:20
Bxxxxxxx                                        True 18/11/2014 08:38:34
Administrator                                   True 03/11/2014 20:32:05

Other parameters worth mentioning:

Search-ADAccount -AccountExpired
Search-ADAccount -AccountDisabled
Search-ADAccount -AccountInactive

Get-Help Search-ADAccount -ShowWindow

Remove .php extension with .htaccess

I've ended up with the following working code:

RewriteEngine on 
RewriteCond %{THE_REQUEST} /([^.]+)\.php [NC]
RewriteRule ^ /%1 [NC,L,R]

RewriteCond %{REQUEST_FILENAME}.php -f
RewriteRule ^ %{REQUEST_URI}.php [NC,L]

Generate a unique id

Why don't use GUID?

Guid guid = Guid.NewGuid();
string str = guid.ToString();

How to get WooCommerce order details

You can get all details by order object.

   // Get $order object from order ID
      
    $order = wc_get_order( $order_id );
      
    // Now you have access to (see above)...
      
    if ( $order ) {
       // Get Order ID and Key
    $order->get_id();
    $order->get_order_key();
     
    // Get Order Totals $0.00
    $order->get_formatted_order_total();
    $order->get_cart_tax();
    $order->get_currency();
    $order->get_discount_tax();
    $order->get_discount_to_display();
    $order->get_discount_total();
    $order->get_fees();
    $order->get_formatted_line_subtotal();
    $order->get_shipping_tax();
    $order->get_shipping_total();
    $order->get_subtotal();
    $order->get_subtotal_to_display();
    $order->get_tax_location();
    $order->get_tax_totals();
    $order->get_taxes();
    $order->get_total();
    $order->get_total_discount();
    $order->get_total_tax();
    $order->get_total_refunded();
    $order->get_total_tax_refunded();
    $order->get_total_shipping_refunded();
    $order->get_item_count_refunded();
    $order->get_total_qty_refunded();
    $order->get_qty_refunded_for_item();
    $order->get_total_refunded_for_item();
    $order->get_tax_refunded_for_item();
    $order->get_total_tax_refunded_by_rate_id();
    $order->get_remaining_refund_amount();
    }

Difference between arguments and parameters in Java

There are different points of view. One is they are the same. But in practice, we need to differentiate formal parameters (declarations in the method's header) and actual parameters (values passed at the point of invocation). While phrases "formal parameter" and "actual parameter" are common, "formal argument" and "actual argument" are not used. This is because "argument" is used mainly to denote "actual parameter". As a result, some people insist that "parameter" can denote only "formal parameter".

How do I create a master branch in a bare Git repository?

A branch is just a reference to a commit. Until you commit anything to the repository, you don't have any branches. You can see this in a non-bare repository as well.

$ mkdir repo
$ cd repo
$ git init
Initialized empty Git repository in /home/me/repo/.git/
$ git branch
$ touch foo
$ git add foo
$ git commit -m "new file"
1 file changed, 0 insertions(+), 0 deletions(-)
create mode 100644 foo
$ git branch
* master

How to add parameters to an external data query in Excel which can't be displayed graphically?

Easy Workaround (no VBA required)

  1. Right Click Table, expand "Table" context manu, select "External Data Properties"
  2. Click button "Connection Properties" (labelled in tooltip only)
  3. Go-to Tab "Definition"

From here, edit the SQL directly by adding '?' wherever you want a parameter. Works the same way as before except you don't get nagged.

paint() and repaint() in Java

The paint() method supports painting via a Graphics object.

The repaint() method is used to cause paint() to be invoked by the AWT painting thread.

Is there a Pattern Matching Utility like GREP in Windows?

An excellent and very fast file search utility, Agent Ransack, supports regular expression searching. It's primarily a GUI utility, but a command-line interface is also available.

How can I declare dynamic String array in Java

no, there is no way to make array length dynamic in java. you can use ArrayList or other List implementations instead.

Trigger back-button functionality on button click in Android

public boolean onKeyDown(int keyCode, KeyEvent event) {
             if (keyCode == KeyEvent.KEYCODE_BACK) {
                     // your code here
                     return false;
             }
         return super.onKeyDown(keyCode, event);
     }

Could not load file or assembly 'System.Web.WebPages.Razor, Version=3.0.0.0

Another option is to update the Microsoft.AspnNet.Mvc NuGet package. Be careful, because NuGet update does not update the Web.Config. You should update all previous version numbers to updated number. For example if you update from asp.net MVC 4.0.0.0 to 5.0.0.0, then this should be replaced in the Web.Config:

    <sectionGroup name="system.web.webPages.razor" type="System.Web.WebPages.Razor.Configuration.RazorWebSectionGroup, System.Web.WebPages.Razor, Version=3.0.0.0, Culture=neutral, PublicKeyToken=31BF3856AD364E35">
      <section name="host" type="System.Web.WebPages.Razor.Configuration.HostSection, System.Web.WebPages.Razor, Version=3.0.0.0, Culture=neutral, PublicKeyToken=31BF3856AD364E35" requirePermission="false" />
      <section name="pages" type="System.Web.WebPages.Razor.Configuration.RazorPagesSection, System.Web.WebPages.Razor, Version=3.0.0.0, Culture=neutral, PublicKeyToken=31BF3856AD364E35" requirePermission="false" />
    </sectionGroup>
  </configSections>

 <host factoryType="System.Web.Mvc.MvcWebRazorHostFactory, System.Web.Mvc, Version=5.0.0.0, Culture=neutral, PublicKeyToken=31BF3856AD364E35" />

<pages
    validateRequest="false"
    pageParserFilterType="System.Web.Mvc.ViewTypeParserFilter, System.Web.Mvc, Version=5.0.0.0, Culture=neutral, PublicKeyToken=31BF3856AD364E35"
    pageBaseType="System.Web.Mvc.ViewPage, System.Web.Mvc, Version=5.0.0.0, Culture=neutral, PublicKeyToken=31BF3856AD364E35"
    userControlBaseType="System.Web.Mvc.ViewUserControl, System.Web.Mvc, Version=5.0.0.0, Culture=neutral, PublicKeyToken=31BF3856AD364E35">
  <controls>
    <add assembly="System.Web.Mvc, Version=5.0.0.0, Culture=neutral, PublicKeyToken=31BF3856AD364E35" namespace="System.Web.Mvc" tagPrefix="mvc" />
  </controls>
</pages>

pandas python how to count the number of records or rows in a dataframe

Simple method to get the records count:

df.count()[0]

Seconds CountDown Timer

int segundo = 0;
DateTime dt = new DateTime();

private void timer1_Tick(object sender, EventArgs e){
    segundo++;
    label1.Text = dt.AddSeconds(segundo).ToString("HH:mm:ss");
}

Are duplicate keys allowed in the definition of binary search trees?

The elements ordering relation <= is a total order so the relation must be reflexive but commonly a binary search tree (aka BST) is a tree without duplicates.

Otherwise if there are duplicates you need run twice or more the same function of deletion!

How to Disable landscape mode in Android?

You can force your particular activity to always remain in portrait mode by writing this in your manifest.xml

 <activity android:name=".MainActivity"
        android:screenOrientation="portrait"></activity>

You can also force your activity to remain in postrait mode by writing following line in your activity's onCreate() method

    @Override
    protected void onCreate(Bundle savedInstanceState) {
        // TODO Auto-generated method stub
        super.onCreate(savedInstanceState);
        setContentView(R.layout.your_layout);
setRequestedOrientation(ActivityInfo.SCREEN_ORIENTATION_PORTRAIT);

}

Switch android x86 screen resolution

To change the Android-x86 screen resolution on VirtualBox you need to:

  1. Add custom screen resolution:
    Android <6.0:

    VBoxManage setextradata "VM_NAME_HERE" "CustomVideoMode1" "320x480x16"
    

    Android >=6.0:

    VBoxManage setextradata "VM_NAME_HERE" "CustomVideoMode1" "320x480x32"
    
  2. Figure out what is the ‘hex’-value for your VideoMode:
    2.1. Start the VM
    2.2. In GRUB menu enter a (Android >=6.0: e)
    2.3. In the next screen append vga=ask and press Enter
    2.4. Find your resolution and write down/remember the 'hex'-value for Mode column

  3. Translate the value to decimal notation (for example 360 hex is 864 in decimal).

  4. Go to menu.lst and modify it:
    4.1. From the GRUB menu select Debug Mode
    4.2. Input the following:

    mount -o remount,rw /mnt  
    cd /mnt/grub  
    vi menu.lst
    

    4.3. Add vga=864 (if your ‘hex’-value is 360). Now it should look like this:

    kernel /android-2.3-RC1/kernel quiet root=/dev/ram0 androidboot_hardware=eeepc acpi_sleep=s3_bios,s3_mode DPI=160 UVESA_MODE=320x480 SRC=/android-2.3-RC1 SDCARD=/data/sdcard.img vga=864

    4.4. Save it:

    :wq
    
  5. Unmount and reboot:

    cd /
    umount /mnt
    reboot -f
    

Hope this helps.

Change color of Label in C#

You can try this with Color.FromArgb:

Random rnd = new Random();
lbl.ForeColor = Color.FromArgb(rnd.Next(255), rnd.Next(255), rnd.Next(255));

How to specify jdk path in eclipse.ini on windows 8 when path contains space

I was facing the same issue but was unable to solve, until I try this:

  1. Please make sure you put -vm
  2. Then press Enter
  3. And then paste C:\Program Files\Java\jdk1.6.0_07\bin\javaw.exe

How to convert string to binary?

This is an update for the existing answers which used bytearray() and can not work that way anymore:

>>> st = "hello world"
>>> map(bin, bytearray(st))
Traceback (most recent call last):
  File "<stdin>", line 1, in <module>
TypeError: string argument without an encoding

Because, as explained in the link above, if the source is a string, you must also give the encoding:

>>> map(bin, bytearray(st, encoding='utf-8'))
<map object at 0x7f14dfb1ff28>

Turn a single number into single digits Python

If you want to change your number into a list of those numbers, I would first cast it to a string, then casting it to a list will naturally break on each character:

[int(x) for x in str(n)]

What do I need to do to get Internet Explorer 8 to accept a self signed certificate?

Unfortunately none of the solutions worked for me. I used Internet Explorer 8 on Windows 7. When I was looking for a solution, I found the settings about login information in the control panel. So I added a new entry under the certificate based information with the address of my server and I chose my prefered certificate.

After a clear of the SSL cache in Internet Explorer 8 I just refreshed the site and the right certificate was sent to the server.

This isn't the solution which I wanted, but it works.

how to run a command at terminal from java program?

You need to run it using bash executable like this:

Runtime.getRuntime().exec("/bin/bash -c your_command");

Update: As suggested by xav, it is advisable to use ProcessBuilder instead:

String[] args = new String[] {"/bin/bash", "-c", "your_command", "with", "args"};
Process proc = new ProcessBuilder(args).start();

How to call python script on excel vba?

I just came across this old post. Nothing listed above actually worked for me. I tested the script below, and it worked fine on my system. Sharing here, for the benefit of others who come here after me.

Sub RunPython()

Dim objShell As Object
Dim PythonExe, PythonScript As String

    Set objShell = VBA.CreateObject("Wscript.Shell")

    PythonExe = """C:\your_path\Python\Python38\python.exe"""
    PythonScript = "C:\your_path\from_vba.py"

    objShell.Run PythonExe & PythonScript

End Sub

How can I check for an empty/undefined/null string in JavaScript?

The following regular expression is another solution, that can be used for null, empty or undefined string.

(/(null|undefined|^$)/).test(null)

I added this solution, because it can be extended further to check empty or some value like as follow. The following regular expression is checking either string can be empty null undefined or it has integers only.

(/(null|undefined|^$|^\d+$)/).test()

How do I navigate to another page when PHP script is done?

if ($done)
{
    header("Location: /url/to/the/other/page");
    exit;
}

Linq with group by having count

Below solution may help you.

var unmanagedDownloadcountwithfilter = from count in unmanagedDownloadCount.Where(d =>d.downloaddate >= startDate && d.downloaddate <= endDate)
group count by count.unmanagedassetregistryid into grouped
where grouped.Count() > request.Download
select new
{
   UnmanagedAssetRegistryID = grouped.Key,
   Count = grouped.Count()
};

Abstraction vs Encapsulation in Java

OO Abstraction occurs during class level design, with the objective of hiding the implementation complexity of how the the features offered by an API / design / system were implemented, in a sense simplifying the 'interface' to access the underlying implementation.

The process of abstraction can be repeated at increasingly 'higher' levels (layers) of classes, which enables large systems to be built without increasing the complexity of code and understanding at each layer.

For example, a Java developer can make use of the high level features of FileInputStream without concern for how it works (i.e. file handles, file system security checks, memory allocation and buffering will be managed internally, and are hidden from consumers). This allows the implementation of FileInputStream to be changed, and as long as the API (interface) to FileInputStream remains consistent, code built against previous versions will still work.

Similarly, when designing your own classes, you will want to hide internal implementation details from others as far as possible.

In the Booch definition1, OO Encapsulation is achieved through Information Hiding, and specifically around hiding internal data (fields / members representing the state) owned by a class instance, by enforcing access to the internal data in a controlled manner, and preventing direct, external change to these fields, as well as hiding any internal implementation methods of the class (e.g. by making them private).

For example, the fields of a class can be made private by default, and only if external access to these was required, would a get() and/or set() (or Property) be exposed from the class. (In modern day OO languages, fields can be marked as readonly / final / immutable which further restricts change, even within the class).

Example where NO information hiding has been applied (Bad Practice):

class Foo {
   // BAD - NOT Encapsulated - code external to the class can change this field directly
   // Class Foo has no control over the range of values which could be set.
   public int notEncapsulated;
}

Example where field encapsulation has been applied:

class Bar {
   // Improvement - access restricted only to this class
   private int encapsulatedPercentageField;

   // The state of Bar (and its fields) can now be changed in a controlled manner
   public void setEncapsulatedField(int percentageValue) {
      if (percentageValue >= 0 && percentageValue <= 100) {
          encapsulatedPercentageField = percentageValue;
      }
      // else throw ... out of range
   }
}

Example of immutable / constructor-only initialization of a field:

class Baz {
   private final int immutableField;

   public void Baz(int onlyValue) {
      // ... As above, can also check that onlyValue is valid
      immutableField = onlyValue;
   }
   // Further change of `immutableField` outside of the constructor is NOT permitted, even within the same class 
}

Re : Abstraction vs Abstract Class

Abstract classes are classes which promote reuse of commonality between classes, but which themselves cannot directly be instantiated with new() - abstract classes must be subclassed, and only concrete (non abstract) subclasses may be instantiated. Possibly one source of confusion between Abstraction and an abstract class was that in the early days of OO, inheritance was more heavily used to achieve code reuse (e.g. with associated abstract base classes). Nowadays, composition is generally favoured over inheritance, and there are more tools available to achieve abstraction, such as through Interfaces, events / delegates / functions, traits / mixins etc.

Re : Encapsulation vs Information Hiding

The meaning of encapsulation appears to have evolved over time, and in recent times, encapsulation can commonly also used in a more general sense when determining which methods, fields, properties, events etc to bundle into a class.

Quoting Wikipedia:

In the more concrete setting of an object-oriented programming language, the notion is used to mean either an information hiding mechanism, a bundling mechanism, or the combination of the two.

For example, in the statement

I've encapsulated the data access code into its own class

.. the interpretation of encapsulation is roughly equivalent to the Separation of Concerns or the Single Responsibility Principal (the "S" in SOLID), and could arguably be used as a synonym for refactoring.


[1] Once you've seen Booch's encapsulation cat picture you'll never be able to forget encapsulation - p46 of Object Oriented Analysis and Design with Applications, 2nd Ed

C#: List All Classes in Assembly

Use Assembly.GetTypes. For example:

Assembly mscorlib = typeof(string).Assembly;
foreach (Type type in mscorlib.GetTypes())
{
    Console.WriteLine(type.FullName);
}

What is a deadlock?

Deadlocks does not just occur with locks, although that's the most frequent cause. In C++, you can create deadlock with two threads and no locks by just having each thread call join() on the std::thread object for the other.

PHP json_encode json_decode UTF-8

For me both methods

<?php

header('Content-Type: text/html; charset=utf-8');

echo json_encode($YourData, \JSON_UNESCAPED_UNICODE);

Function in JavaScript that can be called only once

Here is an example JSFiddle - http://jsfiddle.net/6yL6t/

And the code:

function hashCode(str) {
    var hash = 0, i, chr, len;
    if (str.length == 0) return hash;
    for (i = 0, len = str.length; i < len; i++) {
        chr   = str.charCodeAt(i);
        hash  = ((hash << 5) - hash) + chr;
        hash |= 0; // Convert to 32bit integer
    }
    return hash;
}

var onceHashes = {};

function once(func) {
    var unique = hashCode(func.toString().match(/function[^{]+\{([\s\S]*)\}$/)[1]);

    if (!onceHashes[unique]) {
        onceHashes[unique] = true;
        func();
    }
}

You could do:

for (var i=0; i<10; i++) {
    once(function() {
        alert(i);
    });
}

And it will run only once :)

What is the difference between fastcgi and fpm?

FPM is a process manager to manage the FastCGI SAPI (Server API) in PHP.

Basically, it replaces the need for something like SpawnFCGI. It spawns the FastCGI children adaptively (meaning launching more if the current load requires it).

Otherwise, there's not much operating difference between it and FastCGI (The request pipeline from start of request to end is the same). It's just there to make implementing it easier.

Syntax for async arrow function

Async Arrow function syntax with parameters

const myFunction = async (a, b, c) => {
   // Code here
}

What exactly does stringstream do?

You entered an alphanumeric and int, blank delimited in mystr.

You then tried to convert the first token (blank delimited) into an int.

The first token was RS which failed to convert to int, leaving a zero for myprice, and we all know what zero times anything yields.

When you only entered int values the second time, everything worked as you expected.

It was the spurious RS that caused your code to fail.

Python Pandas counting and summing specific conditions

You didn't mention the fancy indexing capabilities of dataframes, e.g.:

>>> df = pd.DataFrame({"class":[1,1,1,2,2], "value":[1,2,3,4,5]})
>>> df[df["class"]==1].sum()
class    3
value    6
dtype: int64
>>> df[df["class"]==1].sum()["value"]
6
>>> df[df["class"]==1].count()["value"]
3

You could replace df["class"]==1by another condition.

Add new row to dataframe, at specific row-index, not appended?

The .before argument in dplyr::add_row can be used to specify the row.

dplyr::add_row(
  cars,
  speed = 0,
  dist = 0,
  .before = 3
)
#>    speed dist
#> 1      4    2
#> 2      4   10
#> 3      0    0
#> 4      7    4
#> 5      7   22
#> 6      8   16
#> ...

Set formula to a range of cells

Range("C1:C10").Formula = "=A1+B1"

Simple as that.

It autofills (FillDown) the range with the formula.

Send POST parameters with MultipartFormData using Alamofire, in iOS Swift

This is how i solve my problem

let parameters = [
            "station_id" :        "1000",
            "title":      "Murat Akdeniz",
            "body":        "xxxxxx"]

let imgData = UIImageJPEGRepresentation(UIImage(named: "1.png")!,1)



    Alamofire.upload(
        multipartFormData: { MultipartFormData in
        //    multipartFormData.append(imageData, withName: "user", fileName: "user.jpg", mimeType: "image/jpeg")

            for (key, value) in parameters {
                MultipartFormData.append(value.data(using: String.Encoding.utf8)!, withName: key)
            }

        MultipartFormData.append(UIImageJPEGRepresentation(UIImage(named: "1.png")!, 1)!, withName: "photos[1]", fileName: "swift_file.jpeg", mimeType: "image/jpeg")
        MultipartFormData.append(UIImageJPEGRepresentation(UIImage(named: "1.png")!, 1)!, withName: "photos[2]", fileName: "swift_file.jpeg", mimeType: "image/jpeg")


    }, to: "http://platform.twitone.com/station/add-feedback") { (result) in

        switch result {
        case .success(let upload, _, _):

            upload.responseJSON { response in
                print(response.result.value)
            }

        case .failure(let encodingError): break
            print(encodingError)
        }


    }

Cannot open include file: 'unistd.h': No such file or directory

The "uni" in unistd stands for "UNIX" - you won't find it on a Windows system.

Most widely used, portable libraries should offer alternative builds or detect the platform and only try to use headers/functions that will be provided, so it's worth checking documentation to see if you've missed some build step - e.g. perhaps running "make" instead of loading a ".sln" Visual C++ solution file.

If you need to fix it yourself, remove the include and see which functions are actually needed, then try to find a Windows equivalent.

Eclipse Bug: Unhandled event loop exception No more handles

I have a nvidia GPU, and if nView is enabled it happens all the time. Try to disable it.

It seems that eclipse is not very compatible with softwares that override system window management on multi screen.

Hint how to disable nView: http://nviewdesktopmanager.blogspot.com/2011/08/how-to-disable-nview-desktop-manager.html

How to copy data to clipboard in C#

There are two classes that lives in different assemblies and different namespaces.

  • WinForms: use following namespace declaration, make sure Main is marked with [STAThread] attribute:

    using System.Windows.Forms;
    
  • WPF: use following namespace declaration

    using System.Windows;
    
  • console: add reference to System.Windows.Forms, use following namespace declaration, make sure Main is marked with [STAThread] attribute. Step-by-step guide in another answer

    using System.Windows.Forms;
    

To copy an exact string (literal in this case):

Clipboard.SetText("Hello, clipboard");

To copy the contents of a textbox either use TextBox.Copy() or get text first and then set clipboard value:

Clipboard.SetText(txtClipboard.Text);

See here for an example. Or... Official MSDN documentation or Here for WPF.


Remarks:

Peak-finding algorithm for Python/SciPy

There is a function in scipy named scipy.signal.find_peaks_cwt which sounds like is suitable for your needs, however I don't have experience with it so I cannot recommend..

http://docs.scipy.org/doc/scipy/reference/generated/scipy.signal.find_peaks_cwt.html

Efficient way to rotate a list in python

What is the use case? Often, we don't actually need a fully shifted array --we just need to access a handful of elements in the shifted array.

Getting Python slices is runtime O(k) where k is the slice, so a sliced rotation is runtime N. The deque rotation command is also O(k). Can we do better?

Consider an array that is extremely large (let's say, so large it would be computationally slow to slice it). An alternative solution would be to leave the original array alone and simply calculate the index of the item that would have existed in our desired index after a shift of some kind.

Accessing a shifted element thus becomes O(1).

def get_shifted_element(original_list, shift_to_left, index_in_shifted):
    # back calculate the original index by reversing the left shift
    idx_original = (index_in_shifted + shift_to_left) % len(original_list)
    return original_list[idx_original]

my_list = [1, 2, 3, 4, 5]

print get_shifted_element(my_list, 1, 2) ----> outputs 4

print get_shifted_element(my_list, -2, 3) -----> outputs 2